sábado, 6 de maio de 2017

“Para aqueles de nós que acreditam na física, esta separação entre passado, presente e futuro é somente uma ilusão.” (Albert Einstein) O que acha? Einstein se engana?

Sim. Engana-se. Passado, presente e futuro não são ilusões. São posições ao longo do tempo perfeitamente definidas para cada observador. O que pode acontecer é que essas posições não sejam as mesmas para observadores diferentes.

Professor, o que pensa a respeito desta realidade ser uma mera ilusão criada por seres infinitamente superiores para entretenimento individual, como sugere o filme Matrix ?

Uma consideração inteiramente fantasiosa.

Qual a sua opinião a respeito do professor Olavo de Carvalho?

Uma pessoa de ideias completamente obtusas e retrógradas.

Se alguém propusesse hoje a você congelar seu corpo após a morte para que seja possível no futuro te reviver, aceitaria?

Sim, se eu não tivesse despesa com isso, pois dinheiro eu não tenho.

Você acha que algum dia humanos vão conseguir pisar em Marte? Se sim, ate que ano você acha que eles chegariam no planeta vermelho?

Sim, mas não tenho ideia de quando. Acho que isso será possível dentro de uns 50 anos. Mas é só um palpite.

https://ask.fm/wolfedler/answers/135918207517 Mas esse é exatamente o motivo de chamarem de golpe... É tudo feito dentro do processo legal, mas sem base legal pra isso... Ou seja, o impeachment está ocorrendo como deveria, mas sem motivo. Por este motivo é chamado de golpe..

Em meu entendimento isso não é chamado de golpe. Golpe é uma usurpação do poder em confronto com os procedimentos legais. O que está acontecendo é uma manobra política ilegítima, mas legal.

O que eu tenho que saber de matemática para me dar bem no curso de física?

Para começar, tem que ter um excelente traquejo em algebrismo, isto é, manipulações de equações, extração de incógnitas em expressões com frações, radicais e coisas assim. Isso é essencial, bem como usar as identidades algébricas notáveis. Vai precisar de resolver equações e sistemas de equações. Depois tem que conhecer bem funções em geral e, em particular, as trigonométricas, exponenciais e logarítmicas. Tem que ter facilidade em usar as identidades trigonométricas e as propriedades das exponenciais e dos logaritmos. Vai usar muito derivadas e integrais, mas isso se aprende no próprio curso. Tem que saber obter valores de comprimentos e ângulos em figuras geométricas em geral.

Professor, atualmente curso filosofia e técnico em música, pretendo, ao término da minha graduação, tentar ingressar em um conservatório de música, pois então, há alguma junção entre música e filosofia? alguma área de estudo, ou algo do tipo?

Existe a Filosofia da Música e a Estética da Música. Procure ler as obras a respeito de Theodor Adorno.

Quão reais são suas expectativas em relação a si mesmo

Considerando que sou possuidor de insuficiência cardíaca grave minha expectativa real é de morrer em breve.

É possível fazer uma pessoa deixar de ser preconceituosa? Tenho impressão que com jovens é mais fácil de desconstruir preconceitos.

Uma pessoa que já tenha formado sua personalidade e caráter e seja preconceituosa, dificilmente deixará de sê-lo, pois isso já faz parte de sua visão de mundo e a visão de mundo é algo muito arraigado na pessoa. Mas não é impossível mudar, só que com grande dificuldade. Nos jovens é mais fácil, pois eles ainda estão construindo sua visão de mundo e ainda são sensíveis a argumentos capazes de modificar as concepções que estão formando.

http://mobile.opovo.com.br/app/maisnoticias/brasil/2016/04/27/noticiasbrasil,3608279/deputados-aprovam-lei-que-proibe-professores-de-opinar-em-sala-de-aula.shtml o que acha sobre ?

Acho um absurdo. Claro que professores, na qualidade de professores, são moralmente obrigados a apresentar, sempre, os vários pontos de vista existentes sobre qualquer assunto que se aborde e os argumentos dos que os defendem e dos que os atacam. Isso faz parte da honestidade intelectual que o professor tem a obrigação de observar. Mas ele tem todo o direito de ter a sua própria convicção a respeito, apresentá-la e defendê-la, seja qual for, desde que cumpra o que eu disse no começo. Impedir isso é um ultraje à liberdade de pensamento garantida pelo artigo 5 da Constituição Federal.

Professor, sobre o golpe. Eu não vejo golpe algum. O STF não ver golpe. Mas Adolfo Pérez Esquivel, Nobel da paz em 80, discursou no senado dizendo que é golpe. Será que ele está mais informado que os brasileiros? Caso não esteja, como o PT consegue esse tipo de apoio, isso é moralmente justificável?

Mesmo considerando que a Dilma não incorra em nenhum problema que possa levar a seu impedimento de governar, não acho que o processo de impedimento ora em curso se configure em um "golpe", pois está sendo levado dentro da legalidade. A permissão que a Câmara deu para que o processo tramite e que o Senado também poderá dar é um resultado politico e não jurídico. Se essa permissão for concedida pelo Senado, então é que haverá o julgamento.

O formato de alguns orbitais no modelo atômico de Schrodinger assemelha-se a uma área "geométrica" muito parecida com o do símbolo de infinito, sabe me dizer geometricamente, o pq do orbital ser definido assim? Existe alguma propriedade relativa ao formato geométrico? Já que foi obtido por cálculos

A forma dos orbitais atômicos e moleculares não é arbitrária e sim resultante da solução da equação de Schrödinger para a configuração considerada. Ela apresenta o campo de densidade de probabilidade de se encontrar o elétron naquela posição, se se pretender pegá-lo. Em verdade ela significa a densidade do campo de matéria do elétron quando ligado ao átomo, molécula ou rede cristalina.

Você já foi pra Rússia? uma coisa que eu queria saber mas nunca encontrei informação é se soviéticos podiam viajar pros EUA nos tempos da guerra fria e vice-versa?

Podiam, desde que tivessem autorização governamental. Como é normal, mesmo entre o Brasil e os Estados Unidos e a maioria dos países. Só que essa autorização era difícil de ser conseguida.

Professor, eu tento olhar para o céu noturno e ver aqueles pontinhos parados brilhantes que são as estrelas e realmente não consigo imaginar que naquele mesmo momento estão viajando a milhões de anos-luz numa velocidade surreal e não percebemos. Realmente é uma coisa impressionante não acha?

De fato. Veja algumas velocidades:
Velocidade do som no ar: 1.200 km/h
Velocidade de rotação do equador da Terra em relação ao seu eixo: 1.700 km/h
Velocidade de revolução da Terra em torno do Sol: 17.000 km/h
Velocidade do Sol dentro da Via Láctea: 790.000 km/h
Velocidade da Via Láctea em relação à radiação de fundo: 2.000.000 km/h
Velocidade da luz: 1.000.000.000 km/h

É possível estudar História de maneira autodida?

Sim. História é. Porque depende mais de leituras. Mas é bom se valer de vários autores. Todavia há que se inteirar, com base nos programas dos cursos das faculdades, quais as disciplinas que se tem que estudar e qual a bibliografia de referência delas. Em geral os cursos de História, no Brasil, deixam de lado a história dos países do oriente, como China, Índia, Japão, Mongólia e os outros, bem como os povos periféricos do Império, como os lídios, dácios, frígios e outros. É bom estudar essas histórias também.

Você sabe guardar segredo?

Não. Eu sempre falo tudo que vem na minha cabeça, sem censura. Se alguém quiser me dizer algo para que eu guarde segredo, não diga. Da mesma forma que, se tiver algo que deva ser segredo, eu não digo para ninguém. Mas eu mesmo, acabo falando. Não sei guardar segredo mesmo. Meu cérebro não possui censura. E eu não me preocupo a mínima com isso. Para mim tudo pode e deve ser dito para todo mundo saber. Não concebo que algo possa ou deva ser secreto.

Se há extraterrestres seria possível se comunicar por ondas de rádio com eles?

Em princípio, sim. Isto é o que tenta fazer o programa SETI, há vários anos e que gerou o livro e o filme "CONTATO".

Da para ter o mesmo conhecimento de um acadêmico de física estudando em casa de forma autodidata?

É difícil. Estando fazendo o curso se tem contato com os professores e colegas para discutir os assuntos e tirar as dúvidas. Isso é fundamental, porque Física não é um assunto fácil e surgem muitas dúvidas. Além do que, em casa não se tem os equipamentos de laboratório da faculdade e Física também envolve uma parte experimental que é essencial.

o livro Curso de Fisica de Berkeley não segue a tradicional distribuição dos conteúdos encontrados em outros livros. além disso ele omite (ou considera de conhecimento do leitor) alguns assuntos. realmente vale estudar por ele ou é melhor combinar Moyses, Alonso e Feynman?

Depende do seu objetivo. Se é saber Física, o Berkeley é melhor. Se é ser aprovado nas disciplinas de Física Geral, o Alonso & Finn é melhor. O Feynman não serve como livro texto e sim como leitura complementar. No meu entendimento o bom seria estudar pelos três. O Berkeley requer um ensino médio muito bem feito, de preferência através do PSSC, inclusive com seus "Tópicos Avançados" e a parte de laboratório.

Na questão da evolução, quem evolui é quem é mais forte, quem mais se adapta ou quem mais se reproduz?

A evolução biológica se dá em duas etapas: mutação e seleção. A mutação é aleatória e provocada por agentes químicos ou físicos sobre o DNA das células gaméticas. A seleção se faz pelo sucesso adaptativo à sobrevivência do indivíduo, até que produza descendência, em relação ao ambiente externo e interno (flora e fauna interior). Isso não significa que seja, necessariamente, mais forte e sim que consiga sobreviver às doenças provocadas por agentes internos e externos, à predação de outros seres vivos, e a superação das adversidades naturais, como o clima e outras. Isso pode requerer força ou agilidade, resistência e capacidade adaptativa, no que ressalta, especialmente, a inteligência.

Professor, percebo que você, assim como eu, considera o feminismo um movimento importante, porém, o que você acha daquela parcela mais extremista do movimento ?

As pessoas ditas feministas radicais não são feministas, são femistas. O femismo é a concepção de que as mulheres devem gozar de privilégios em relação aos homens. O feminismo, ao contrário, considera que não devam haver privilégios de espécie alguma nem para homens nem para mulheres. Isto é, que tanto homens quanto mulheres têm que ter os mesmos direitos e os mesmos deveres. Gozar das mesmas oportunidades e arcar com as mesmas responsabilidades. O femismo é o reverso do machismo, sendo uma concepção tão abominável quanto ele.

Meu querido, acabei me deparando aqui com um experimento, relatando oque chamam de "o peso da alma", onde após o exato momento da morte o corpo perde algumas gramas. O que seria esse peso perdido após a morte?

Se isso é mesmo constatado, certamente que não é a alma, pois esta, caso exista, não seria nenhuma entidade física e, portanto, não possuiria propriedades físicas, como massa, extensão, localização, velocidade, energia, carga elétrica e outras.

Você precisa sempre partir de um pressuposto racional para atingir uma máxima filosófica ou a conclusão do seu pensamento filosófico?

Não. Muito frequentemente as conclusões filosóficas partem de constatações empíricas, às quais se aplicam a reflexão e o raciocínio para levar a uma afirmação que estatue algum princípio. Por exemplo, o princípio (equivocado) de que todo evento seja efeito de alguma causa, adveio da observação empírica das ocorrências do mundo acessíveis aos sentidos humanos antes que se tivesse acesso à realidade atômica e sub-atômica. Após esse acesso e a constatação da existência de eventos incausados, tal princípio perdeu a sua validade. Em verdade, quase toda a Filosofia é construída sobre a observação da realidade e dos fenômenos do mundo, não só natural, mas social e cultural.

Qual a relação das religiões coma metafísica?

As concepções religiosas são metafísicas, pois não se referem a entidades nem a fenômenos naturais. Mas a Metafísica não cuida apenas da realidade e dos fenômenos sobrenaturais. Aliás, isso é só um detalhe dentro da abrangência da Metafísica. A Metafísica cuida dos conceitos abstratos, isto é, que não se referem a entidades e ocorrências concretas do mundo natural. Mas a maior parte das abstrações não se refere a nada sobrenatural. A Metafísica cuida de categorizar a realidade e estudar a situação e as relações entre essas categorias. Também busca interpretar o significado e o sentido de tudo, bem como estudar as propriedades e o estado das diversas entidades, classificando-as. A Metafísica é o núcleo central de toda a Filosofia. Então ela cuida, também, das questões pertinentes às religiões, como a existência ou não de deuses e espíritos, suas propriedades (caso existam) e suas relações.

Existe, de fato, o anti-conservadorismo?

Não tenho conhecimento de que exista algum movimento estabelecido especificamente dedicado a combater o conservadorismo. O que sei é que existem pessoas que não são conservadoras.

Ao meu ver, o bacharelado de filosofia é praticamente uma licenciatura, que não dá direito do graduado de dar aula, apenas isso... Continua formando apenas "entendidos de filosofia", professores das ideias já existentes, e não filósofos em si... Concorda? Como seria o curso ideal, ao seu ver?

Concordo plenamente. Isso é o que eu sempre venho dizendo e contestando os cursos de Bacharelado em Filosofia, da mesma forma que em Letras. O que falta, nesses dois cursos, são disciplinas e oficinas de "Criação Filosófica" e "Criação Literária". Disciplinas onde se aprenda a filosofar, a desenvolver idéias, a raciocinar. Do mesmo modo que um cientista aprende a fazer ciência, a pesquisar, a descobrir coisas novas nas diferentes ciências. O filósofo tem que se debruçar sobre a realidade e, calçado no conhecimento do que outros filósofos, das variadas correntes, já disseram sobre o tema, elaborar sua proposta de interpretação e explicação filosófica do assunto. Isso é filosofar. Não apenas assimilar o que outros já disseram sobre qualquer assunto. Isso é que falta se fazer nos cursos de Filosofia. No caso de Letras, a questão é se ter disciplinas em que se aprenda a elaborar um romance, uma novela, um conto e as várias formas de poesia. Além das oficinas em que os alunos se dediquem a escrever romances, novelas, contos, poesias. Como os alunos de música e de artes plásticas fazem em seus campos.

Diria que é "impossível" conseguir acompanhar um bacharelado de física, ou de matemática, sem saber quase nada dos respectivos assuntos à nível do Ensino Médio? Ou com bastante esforço, é possível aprender os assuntos nas "revisões aprofundadas" do curso universitário, e acompanha-lo sem problemas?

É possível fazer um bacharelado em Física não sabendo a física do Ensino Médio sim. Mas "com problemas" e não "sem problemas". As revisões não suprem satisfatoriamente a falta de conhecimentos do Ensino Médio, pois são rápidas e resumidas, sem o necessário tempo de assimilação de cada conteúdo que acontece quando ele todo é visto ao longo de três anos.

Para cursar bacharelado em física, é necessário saber alguma coisa da física do ensino médio? Não sei nada da física do EM, tenho algumas dificuldades porque não acompanhei o EM direito e estou caminhando para ter uma boa formação matemática com o Kumon

Para cursar bacharelado em física, é necessário saber alguma coisa da física do ensino médio? Não sei nada da física do EM, tenho algumas dificuldades porque não acompanhei o EM direito e estou caminhando para ter uma boa formação matemática com o Kumon

Quais os pontos negativos de acreditar em alguma pseudociência?

Se as pseudociências levassem a resultados válidos, mesmo que não comprovados, elas poderiam ser aceitas, como o é o saber dito "vulgar". Mas elas, mesmo pretendendo ter uma fundamentação científica (que não possuem), levam a conclusões errôneas, não verificadas pelas ocorrências. Daí o seu grande perigo. Confiar nelas é algo extremamente perigoso.

Já leu algum livro do Mario Bunge, recomenda? Sei pouco sobre ele, mas parece que ele fez contribuições significativas para a ciência e filosofia.

Comece por seu livro "Física e Filosofia", depois "Teoria e Realidade", "Ciência e Desenvolvimento", "Epistemolotigia", "Ética e Ciência", "Causalidade", "Intuição e Razão", "Racionalidade e Realismo", "Dicionário de Filosofia". São excelentes livros.

Ernesto, é possível explicar precisamente o porquê de a geometria euclidiana não funcionar em um universo curvo?

O fundamento da geometria euclideana é o quinto postulado, que diz que por um ponto externo a uma reta só existe uma paralela a ela. Em espaços curvos isso não acontece, como pode ser verificado experimentalmente, considerando o conceito de reta substituído pelo de geodésica. Nos positivamente curvos as paralelas não existem e nos negativamente curvos o número de paralelas por um ponto é infinito.

Tecnicamente, para você ser comunista você precisa ser materialista. O materialismo renega a metafísica. A base de todas as religiões é a metafísica. Logo, ser marxista significa não ser religioso, acreditar em espíritos, deuses e etc. Não sei quanto ao anarquismo.

Não. É perfeitamente possível se ser comunista crendo em Deus, na alma e em uma realidade sobrenatural. Comunismo é uma concepção econômica, segundo a qual os meios de produção e prestação de serviços, bem como o produto dessa produção e desses serviços não são de propriedade de pessoas ou grupos em particular, nem de nenhum estado ou governo, mas de todos os envolvidos no processo. Isso não tem nada a ver com acreditar ou não em deuses. O marxismo é que é uma concepção materialista do comunismo. Mas o comunismo não precisa ser marxista. Outra coisa: o materialismo, atualmente encarnado no fisicalismo, já que a realidade física não é composta apenas de matéria, não renega a Metafísica. Ele renega a realidade sobrenatural. Mas a Metafísica não cuida do sobrenatural, mesmo que também considere se possa ou não existir. A Metafísica cuida daquilo que não é físico, isto é, das abstrações. E abstração não é o mesmo que sobrenatural. Quanto ao anarquismo, é uma concepção política, que também não inclui a ateísmo como requisito. Anarquismo é a concepção de que a sociedade seja estabelecida sem a existência de governo e nem do estado político que o abriga. Os três conceitos, comunismo, anarquismo e ateísmo são independentes e cada um pode ocorrer sem os outros dois. De minha parte, eu abraço os três.

Para um pessoa ser anarquista/comunista ela tem que ser ateia?

Não. São concepções independentes. Tanto se pode ser anarquista ou comunista acreditando em Deus, como ser ateu e não ser anarquista nem comunista.

Por que não gosta quando te chamam de "senhor"?

Porque sou extremamente igualitário e considero que todas as pessoas possuem o mesmo estatuto na sociedade, não sendo ninguém superior nem inferior a ninguém, apenas executando funções diferentes. Assim eu não chamo ninguém de "senhor" e prefiro que não me chamem também. Mas não me incomodo se chamarem. Não atribuo valor a ninguém por sua dita "importância" em termo do papel que exerça, do dinheiro que possua, do poder que detenha, do conhecimento que abarque, da investidura que seja revestido. Para mim o valor de uma pessoa está em seu bom caráter, independentemente de outros aspectos. Assim eu prezo a todos e só desprezo a quem tenha mau caráter.

Para ser mais especifico nessa pergunta, o que são flutuações quânticas?

São variações aleatórias no estado quântico de um sistema. Isso se aplica, especialmente, ao caso do campo do vácuo, cujas flutuações quânticas dão azo ao surgimento de pares de partícula e antipartícula.

Professor, o que você pensa a respeito de cursar Ciências Econômicas? Devo ser bom em cálculos?

A Matemática requerida para as Ciências Econômicas é elementar. O que mais se precisa é ter uma compreensão clara dos mecanismos e fatores causadores dos eventos econômicos e quais as suas consequências. Isso depende de muito conhecimento geral sobre economia, sobre a história da economia e de se ter uma forte intuição econômica. Isso é que é o mais importante. Os cálculos, em economia, são secundários, mesmo que sejam necessários. É importante saber que Economia não é uma ciência exata e sim uma ciência social. Portanto tem uma grande interação com sociologia, política, história e geografia. Além de psicologia.

Como a física/mecânica quântica implica na inexistência de deus? Tem haver com as flutuações quânticas não tem?

A Física, em especial a Mecânica Quântica, bem como toda e qualquer ciência, absolutamente, não implicam na inexistência de Deus. Elas, simplesmente, não levam essa existência em consideração e fornecem indícios da inexistência, uma vez que os fenômenos são explicados sem supor tal existência. Mas elas não obstam que possa existir.

Olha a situação que a Venezuela passa. Comunismo é isso aí e vc apoia esse regime ridículo.

Não. Mas o regime da Venezuela não é comunista, é socialista. É muito diferente. No comunismo a posse dos meios de produção e serviços, bem como dos resultados dessa produção e desse serviço não é do governo, é das pessoas. Ainda não existiu um regime comunista no mundo. O que se chama de comunismo não é comunismo, é socialismo de estado, que é algo péssimo.

Sobre os dicionários de filosofia, professor

Possuo e consulto sempre os seguintes:
Lalande, Abbagnano, Huisman (de obras e de filósofos), Oxford, Ferrater Mora, Comtr-Sponville, João Branquinho, Jaqueline Russ, Mário Bunge, Japiassú.

Você acha que devemos ser vaidosos com nossa inteligência ?

Não. Acho que a vaidade, a respeito do que quer que seja, é uma completa idiotice.

Para conseguir resolver um problema de matemática, é preciso ser bom em interpretar textos?

Claro que sim. Com problemas de Física e de qualquer assunto. Se não souber interpretar textos não vai entender o que se pede. Tem que ter bom vocabulário também. Além de, é claro, ter o conhecimento específico do assunto de matemática ou física a que o problema se refere.

Shakespeare ou Dostoiévski?

Gosto de ambos, mas prefiro Dostoiévski.

Pesquisas recentes sugerem que cachorros são empáticos quando seus donos estão tristes ou passando por uma situação difícil. Você está 100% convicto que cachorros não pensam, mesmo por mínimo que seja ?

Eu tenho certeza de que cachorros e vários outros animais pensam sim. Tenho experiência pessoal de convívio com vários cachorros ao longo da vida, bem como gatos. Atualmente, em minha cada, temos dez cães e dois gatos. Sempre tive cães, desde criança e sou muito afeiçoado a eles e interajo com eles. Sei que pensam. Percebo facilmente isso. E vejo que muito animais pensam também. Só não falam articuladamente. Mas falam pela entonação de seu latidos ou outras formas de emitirem sons.

Ernesto, estava pensando em cursar filosofia, mas tenho medo das faculdades apenas te ensinarem a ser professor e não a pensar verdadeiramente como um filosofo. O que você acha do curso?

Realmente os cursos de Filosofia no Brasil apenas formam "entendidos em Filosofia" e não filósofos. Isso acontece, também, com os cursos de Letras, que formam "entendidos em Literatura", mas não literatos. Uma lástima. É muito difícil mudar essa realidade. Os professores, inclusive, reprimem o livre pensamento, as contestações à linha filosófica que cada faculdade segue. Todavia é possível, pelo esforço pessoal, superar essa limitação e se tornar um verdadeiro filósofo. Mas tem que dar muito murro em ponta de faca. Claro que, liminarmente, há que se ser um excelente "entendido em Filosofia", para poder começar a pensar em ser filósofo. Isto é, tem-se que ser um aluno muito acima da média, ou seja, um aluno "brilhante". Que seja capaz, nas avaliações, de dar conta de tudo o que os professores querem e ir além, com sólidos argumentos, depois de demonstrar que sabe o que se deseja que seja sabido, contestar esse saber. Não é para qualquer um. Mas não é impossível.

O que acha de Carl Gustav Jung, professor?

Acho que ele deturpou inteiramente as ideias de Freud ao colocar a alma espiritual dentro do contexto da psicologia. Isso é indesculpável. Psicologia é um capítulo da neurologia e não tem nada de alma no meio. É uma ciência natural, fisiológica, materialista (ou melhor, fisicalista). Por outro lado ele introduziu conceitos inteiramente despropositados, como "inconsciente coletivo" e outros. Mas Freud também cometeu muitos equívocos e suas propostas carecem de embasamento, sendo muitas, meramente, opiniões dele. Como sua interpretação dos sonhos.

Cotas nao é uma maneira de tratar desigualmente o desigual, não

É. Mas é uma maneira equivocada. Porque possibilita a premiação de quem não tenha merecimento. Isso está errado, mesmo que seja uma postura esquerdista. Não é possível se conceder um certificado de proficiência para exercer alguma atividade profissional, que é o que as universidades fazem, para quem não tenha provado que adquiriu capacidade para merecê-lo. Isso não depende de se ser rico ou pobre, preto ou branco, índio ou descendente de europeu, mulher ou homem, crente ou ateu, esquerdista ou direitista. A competência profissional depende apenas do saber. E quem não dominá-lo não pode ter a licença para exercer a profissão. Independentemente do que seja. O que resolve é colocar a educação pública no mesmo nível de qualidade da educação privada, para que os pobres, que se valem dela, tenham a mesma capacidade dos ricos. Para tal é preciso que o governo invista na qualificação do magistério e seja rigoroso na exigência para aprovação desde os níveis mais elementares de escolaridade. Só se não se permitir que alunos que não saibam nada continuem a progredir os estudos é que se terá uma população devidamente bem formada em termos da escolaridade que é preciso ter para se fazer um curso superior. O sistema de cotas permite que pessoas com menor qualificação tenham acesso à universidade, forçando-a a rebaixar o seu nível de exigência e, daí, formando profissionais de menor competência. Isso é péssimo para o bem do mundo. Em minha opinião dever-se-ia extinguir a educação privada e tornar toda ela pública, de modo que, não importa se se seja rico ou pobre, só se tem a escola pública para estudar. Então as pessoas ricas, que têm poder, vão fazer com que a escola pública seja de qualidade, pois é nela que seus filhos vão estudar. Por outro lado isso também é ótimo para permitir o convívio entre ricos e pobre num mesmo ambiente, para que todos percebam que são humanamente iguais.

Você concorda que se uma pessoa é arrogante, ela certamente não aprendeu nada da vida? Minha psicoterapeuta comentou isso, e devido à o que ando percebendo de certas arrogâncias, me parece uma premissa pertinente.

Não concordo. Uma pessoa arrogante, certamente aprendeu muita coisa na vida. Mas não aprendeu a ser cortês, gentil, educada, solícita, compreensiva, solidária, cooperativa, generosa, prestativa, amiga, simpática, ter empatia e tudo o que promove a união entre as pessoas e um convívio humano harmônico e fraterno.

Escolhe um trabalho de que gostes, e não terás que trabalhar nem um dia na tua vida. ~ Confúcio. (Comente)

É isso mesmo. A gente precisa trabalhar naquilo que gosta, como eu. Assim não se faz distinção entre o trabalho e o resto da vida. Para mim, estou trabalhando todos os momentos da vida e, naqueles em que estou especificamente dedicado ao trabalho, continuo vivendo minha vida pessoal normalmente. E como usar o computador. Não distingo entre estar usando de modo utilitário ou como lazer. Meu trabalho é meu lazer. E, no lazer, continuo trabalhando, porque estou sempre pensando em educação, ciência, filosofia, arte, cultura, que são os temas a que me dedico ao trabalhar e, também, são os temas a que me dedico em meu lazer. São os temas que povoam meu pensamento o tempo todo.

Mas um elétron não emite somente a cor que absorveu? Por exemplo, se um elétron absorver o fóton de frequência correspondente a cor vermelha, ele não emitirá esse mesmo fóton novamente após decair nos níveis de energia? O mesmo não vale para os átomos? Como podem emitir infravermelho?

Não. Dependendo dos valores dos níveis de energia, ao absorver um fóton, um átomo pode ter um de seus elétrons excitados mais de um nível de energia acima do que estava e, ao decair, pode fazê-lo pela emissão de vários fótons, correspondentes ao decaimento gradativo entre os níveis até voltar ao fundamental. Além do mais, na matéria condensada, diferentemente dos gases, a absorção de um fóton pode excitar não apenas um átomo, mas um grupo deles, uma vez que são ligados. Então tem-se que considerar os níveis de energia do conjunto, que faz com que eles se tornem muito mais próximos e menos diferenciados em energia do que os níveis de um átomo isolado. Isso acontece, mesmo, para apenas uma molécula de vários átomos.

Você se considera um hedonista?

Não, mas me considero um epicurista dialeticamente sintetizado com um estoico. O epicurismo difere do hedonismo no fato de que este busca a realização e o significado da vida no gozo dos prazeres, enquanto aquele o faz na fruição da felicidade, que também inclui o gozo de prazeres, mas de um modo virtuoso. O epicurismo é eudemonista.

O que o senhor pensa acerca da pena de morte?

Sou total e radicalmente contra. Por três razões. A primeira é que não permite a correção de injustiças se um inocente for condenado. A segunda é que não propicia a recuperação do criminoso, que é um dos três objetivos da penalização. A terceira é que se é errado matar, esse erro também é cometido se essa morte for aplicada oficialmente pelo estado. Os três objetivos da penalização são afastar o criminoso do convívio social, para proteger a sociedade, aplicar um castigo que desincentive o crime e recuperar o criminoso para retornar ao convívio social.

"Devemos tratar igualmente os iguais e desigualmente os desiguais, na medida de sua desigualdade" O que acha desse texto? faz sentido?

Faz, mas está incompleto. Enquanto há desigualdade, há que se tratar desigualmente os desiguais, para não se cometer injustiças. Mas é preciso, paralelamente, travar uma luta constante e com muito empenho, para que as desigualdades sejam abolidas. Não as desigualdades pessoais, que são inevitáveis, mas as desigualdades de direitos e oportunidades que a sociedade impõe às pessoas. Essas têm que ser abolidas.

Quais áreas do conhecimento você acha que, estudadas seriamente, possuem maiores chances de levar uma pessoa ao ateísmo ? penso em biologia (principalmente evolução), história (p.história da religião), física, cosmologia, psicologia (p.cognitivista e evolutiva) e filosofia. Acrescenta ou tira algo?

Só falta um assunto: Religião comparada. Juntando com o que você citou temos os temas que levam a pensar na inexistência de Deus e, exatamente, os assuntos que eu mesmo estudei, e bastante, e me levaram à conclusão da inexistência de Deus.

Os budistas reverenciam algum deus?

Não. O Budismo é uma religião ateísta. Mas que considera que existem espíritos.

Uma camisa preta por exemplo, absorve todas as cores da luz incidida? Mas se é "preto", por que os átomos ou elétrons que absorvem os fótons não saltam de nível de energia e liberam a energia que absorveram?

Em um corpo de superfície negra, os átomos absorvem os fótons e, depois, decaem sim. Só que não decaem de volta diretamente para o mesmo nível em que estavam e sim em uma escada de saltos de menos energia, na faixa do infravermelho, que é absorvido pelos átomos e moléculas como um todo, provocando um aumento da agitação térmica.

Você considera o esporte importante? Na sua opinião, que valores o esporte deve passar?

Como uma forma de se fazer exercícios físicos o esporte é bom. Como uma forma de competição não. Sou contra competições com vencedores e perdedores. Acho que ninguém deve desejar derrotar ninguém. Todos devem se ajudar para que cada um supere a si mesmo em suas limitações. Mas sem disputa com vencedores e perdedores. Acho que o esporte deveria se modificar para se tornar um espetáculo de demonstração de força e agilidade, bem como habilidade, sem competição, sem vencedores. Como, por exemplo, uma escalada de montanha, em que o que conta é que todos se auxiliem para que todos alcancem o topo.

É desaconselhável cursar um doutorado em uma área diferente do mestrado e, um mestrado, em uma área diferente da graduação? Por exemplo, se graduar em Filosofia, ter um mestrado em Ciência Política e um doutorado em Economia? Para se tornar um bom pesquisador, é necessário que siga apenas uma área?

Não precisa fazer graduação, mestrado e doutorado na mesma área não. Inclusive é bom fazer em áreas diferentes, desde que tenham alguma correlação. Isso faz com que a pessoa tenha uma visão mais abrangente. Certamente que vai em sentido contrário ao que se considera normalmente, ou seja, que o doutorado seja um afunilamento do mestrado e que este seja um afunilamento da graduação, levando a pessoa a uma super-especialização. Não concordo com essa visão. Para mim é bom que a pessoa, pelo contrário, tenha uma visão bem amplificada e não restrita. Mas nem todos concordam comigo.

Como asseguras que a Terra sabe que gostas dela?

Ela não sabe e nem tem como saber, pois a Terra não é um ser consciente.

Professor Ernesto, como faz para descobrir o quoeficiente de inteligência? Ql o seu?

Submetendo-se a testes, inclusive disponíveis na internet. Visite o site:
http://www.qi-inteligencia.com
Os testes de QI que já fiz indicaram valores que variam entre 135 e 148. Acho razoável uma média de uns 141.

Vou doutorar em neurociência. O que acha da área?

Espetacular. Vê se desenvolve um método de imageria cerebral que consiga interpretar, pelo escaneamento, o conteúdo dos pensamentos, sejam evocações, sejam associações, sejam juízos, sejam raciocínios, sejam emoções, sejam sentimentos, sejam volições.

Quer-se, a qualquer custo, aumentar a participação de mulheres na política. você concorda que se criem vagas exclusivas para mulheres?

Não. Eu acho que a participação das mulheres deve ser sempre equivalente à de homens em toda e qualquer atividade. Mas não acho que isso tenha que ser feito de modo forçado. Tem que acontecer naturalmente, pela insistência das mulheres em ocuparem seus lugares e pela compreensão dos homens de que a humanidade é simétrica em relação aos papéis dos sexos. Do mesmo modo que a participação de quaisquer outros grupos, como raciais, religiosos, econômicos e assim por diante (como rurais e urbanos).

Você concorda que pessoas pobres que não podem bancar uma educação de qualidade, um bom plano de saúde e uma vida confortável para uma criança deveriam evitar ao máximo ter um filho?

Eu acho que toda pessoa tem um desejo muito profundo, entranhado em sua natureza humana, de ter, pelo menos, um filho. Para mim, essa questão da pobreza tem que ser solucionada por meio do comunitarismo, isto é, as pessoas não viverem em habitações monofamiliares e sim em habitações grupais, em que várias famílias compartilham todos os serviços, havendo um só refeitório, uma só lavanderia, um só vestuário, algumas salas de lazer, mas não uma para cada família. Só os quartos de dormir seriam específicos para cada casal que quiser manter a monogamia. Os solteiros podem dormir em aposentos coletivos maiores, havendo quartos a mais para os que quiserem fazer sexo. Desta forma a economia será brutal. O grupo, se for razoável, pode fazer um plano de saúde coletivo (se o governo não resolver, como deve, acabar completamente com a medicina privada, bem como com a educação privada). De qualquer modo, ter mais de um filho não é bom. Mas pretender que não se tenha nenhum, para mim, vai contra a natureza humana.

Comente: "Cada um por si e Deus por todos". Provérbio Português

Provérbio sem nenhum fundamento, uma vez que Deus não existe. Mesmo a primeira parte não é válida. Cada um não deve ser apenas por si, mas por todos. Sendo assim, todos ajudando uns aos outros, a condução da vida se torna muito mais fácil e amena. Isso é que é preciso. Colaboração, compartilhamento, solidariedade. Nada de Deus.

Ernesto, o que acha da Polícia Federal? Meu avô é militar, mas nunca tive vontade de seguir a carreira ou algo similar. Tenho 17 anos e, até então, cursaria Engenharia elétrica. Agora, quero cursar Direito. O que acha da instituição citada?

Para mim é uma atividade de que eu jamais gostaria de participar, mesmo reconhecendo o seu valor. Mas isso é gosto meu. Cada pessoa tem que fazer aquilo que gosta na vida. Se você gosta de ser policial, então seja. Se gosta de ser advogado, seja. Se gosta de ser engenheiro, seja. O que importa é se sua atividade vai ser uma em que você frua prazer em exercer.

Professor você já leu mais ou menos quantos livros de matemática e física?

Já li uns três mil livros na vida. Mas, em geral, os livros de Matemática e Física não são para serem lidos e sim consultados para estudos. Isso eu já consultei mais de mil.

Ernesto, qual livro recomenda para geometria diferencial, tendo uma base matemática bem rigorosa(como em um curso de matemática) e sendo suficiente também como base para relatividade geral?

Sugiro os livros:
Differential Geometry - Heirich Guggenheimer - Dover;
Introduction to Differential Geometry - Abraham Goetz - Addison Wesley.
Também é interessante ver:
Introduction to Differentiable Manifolds - Louis Auslander & Robert McKemzie - Dover.
Em geral, os livros de Relatividade Geral abordam a matemática necessária.
Vai ser preciso estudar Cálculo Tensorial. Veja estes:
Tensor Analysis - I.S. Sokolnikoff - Wiley
An Introduction to Tensor Calculus - Jacques Mercier - Wolters-Noordhoff
Tensor Calculus - Synge & Schild - Dover
An Introduction to Tensor Calculus, Relativity and Cosmology - D.F. Lawden - Wiley

Professor, fui ter a dúvida de propagação, na resolução da eq. de Schrodinger linear independente do tempo, onde dizia q a parte da solução Cexp(jkx) se propagava na direção positiva de x. Mas nesses casos independentes do tempo não sei o sinal de wt, como pode se determinar a direção da propagação?

Na equação independente do tempo não se tem sentido de propagação, mas apenas uma fotografia instantânea da onda. Para se ter o sentido de propagação é preciso haver, também, a parte dependente do tempo. O sentido será das ordenadas crescentes se os sinais dos termos espacial e temporal forem contrários.

Estou com uma dúvida em relação ao sentido de propagação de uma onda na notação exponencial, quando se trabalha no domínio espacial. Faço engenharia elétrica, e os livros de eletromag. na engenharia trazem a propagação no sentido positivo de x como exp(-jkx), mas nos livros de física está exp(jkx).

Você tem que considerar a fase completa da onda, que pode ser kx + wt, kx - wt, wt + kx ou wt - kx, dependendo da escolha. As que têm sinal menos se propagam no sentido positivo dos x e as que têm sinal mais no sentido negativo. Isso pode ser visto considerando que, para a fase ser constante (mesmo ponto da figura da onda), se o sinal for positivo, é preciso que x diminua quando t aumente e, quando o sinal for negativo, é preciso que x aumente quando t aumente.

Professor quando você falou que abomina a falta de pragmatismo e idealismo. Isso seria a falta de objetivos, sonhos e ações para realiza-los? Ou seja, os jovens dessa geração meio que não sabem para onde estão indo? O que querem? Claro, há exceções como você disse.

Eu não disse que abomino a falta de pragmatismo e idealismo. Eu disse que a abomino a falta de idealismo e o pragmatismo. Não a falta de pragmatismo. Pragmatismo é a postura de se considerar válido apenas o que resulte em resultados práticos, que tenha utilidade prática. Isso é que eu abomino. Não vejo que o valor de algo se prenda a sua utilidade. Muita coisa extremamente valiosa não tem utilidade nenhuma. Como as artes. Ou o estudo da Cosmologia. Também abomino a falta de idealismo, isto é, de não se querer dedicar a nada que não traga resultados práticos ou alguma compensação financeira. Não gosto de pessoas que não se dediquem ao que lhes dê trabalho, ocupe tempo, consuma custos e sacrifícios mas não resulte em proveito pessoal. Gosto de pessoas altruístas, que se dedicam ao bem dos outros e do mundo sem recompensa nenhuma, pelo contrário, com dispêndio de tempo, energia, dinheiro e esforços a fundo perdido. Isso é que vejo que falta na juventude: idealismo, altruísmo.

Boa noite professor. A minha dúvida é a seguinte, como nos sabemos que o Sol tem um nucleo e tbm as outras estrelas que esta distante de nos? E como sabemos que ela vai se tornar uma gigante vermelha? abc!!!

A existência de um núcleo solar está mostrada nos artigos abaixo:
https://en.wikipedia.org/wiki/Solar_core
http://www.nobelprize.org/nobel_prizes/themes/physics/fusion/
http://www.phy6.org/stargaze/Sun7enrg.htm
A evolução estelar nestes artigos:
https://en.wikipedia.org/wiki/Stellar_evolution
http://umich.edu/~gs265/star.htm
http://www2.le.ac.uk/departments/physics/research/xroa/astronomical-facilities-1/educational-guide/copy_of_stars/stellar-evolution
https://www.aavso.org/stellar-evolution
http://www.stellar-database.com/evolution.html
http://www2.le.ac.uk/departments/physics/research/xroa/astronomical-facilities-1/educational-guide/copy_of_stars/stellar-evolution
http://astronomy.swin.edu.au/cosmos/S/stellar+evolution
http://cosmos.phy.tufts.edu/~zirbel/ast21/handouts/StellarEvolution.PDF

Qual a sua opinião sobre o uso da clonagem como uma alternativa para salvar animais ameaçados de extinção?

Excelente ideia. Tem que ver se os clones vão ser capazes de se reproduzir.

O Físico precisa necessariamente trabalhar com ensino? Pode trabalhar em industrias?

Não precisa trabalhar no ensino. Pode trabalhar em indústrias e em institutos de pesquisa.

Professor, o que acha dessa frase do Schopenhauer "Quem ama sente uma enorme desilusão depois de finalmente chegar ao prazer. E, surpreso, vê que aquilo que tanto desejou traz o mesmo que qualquer outra satisfação sexual , e assim não encontrará muita vantagem em amar." ?

Ela revela o sentimento de uma pessoa que nunca amou. O sexo, com envolvimento amoroso, é muito mais gratificante do que sem. Não resta dúvida e quem já experimentou as duas situações pode facilmente atestar.

Uma dúvida, se eu tenho duas caneta, uma se movendo e outra parada, a caneta parada vai estar em movimento em relação a caneta em movimento mesmo estando parada?

Claro que sim. Quando você diz que uma delas está parada, você diz que ela não se move em relação, por exemplo, ao solo do planeta Terra. Mas ela está se movendo, com o planeta Terra, em sua órbita em torno do Sol. Todo movimento é relativo a um referencial.

A evolução dotou todos os mamíferos com orgãos indispensáveis: cérebro, coração, pulmão, sexo, olhos, etc. Nem todos têm rabo, e apenas um tem a razão. Será que para a natureza mais vale um rabo balançando, que um cérebro pensando?

Não é apenas uma espécie de animal que possui raciocínio. A humana o possui em grau superlativo, mas há várias que também o possuem em grau menor. Inclusive não vertebradas, como os cefalópodes. Não há diferença ontológica entre um ser humano e os outros animais. Tudo é uma questão de grau. Outras espécies, já extintas, também possuíam o mesmo grau de complexidade neurológica que possibilita a razão, como os neandertais. E outras ainda poderão surgir, ao longo da evolução. Bem como em outros planetas que possuam seres vivos. A natureza não confere valor a nada. Tudo o que surge, surge por acaso e permanece se for vantajoso para o sucesso em sobreviver. As caudas são resquícios das nadadeiras caudais dos peixes, que eram essenciais para a propulsão na água (note que os peixes executam a propulsão por movimentos sinuosos horizontais enquanto os mamíferos aquáticos o fazem por movimentos sinuosos verticais, daí as nadadeiras caudais dos primeiros serem verticais e dos segundos serem horizontais). Todavia as caudas se revelaram muito úteis para os primatas arborícolas, mas não para os de chão, como gorilas e homens.

Matemática ou física? Qual conselho você daria para quem está em dúvida de qual cursar? Em relação ao bacharelado.

Na dúvida, faça Física, que você também estudará muita Matemática. Além do mais a Física é mais ligada à realidade objetiva do mundo. Mesmo usando abstrações matemáticas para descrevê-la, requer uma correspondência com os fatos realmente ocorrentes.

Professor quais as principais críticas você tem para os jovens dessa geração? O que você acha que deu errado?

Minha maior crítica, de um modo geral, é a falta de idealismo e o pragmatismo. Abomino isso. Também critico a fobia por tudo que seja complicado, trabalhoso, difícil, cansativo, demorado, custoso. São as coisas assim que resolvem. Tudo o que é fácil, ameno, simples, rápido, não resolve nada. Outra coisa, ainda, é a falta de curiosidade, de desejo de saber, sem se preocupar com a utilidade. A falta de curtição em relação à arte mais erudita também considero que seja um ponto a ser criticado na nova geração. Há exceções, é claro.

Comente: "Não cuideis que vim trazer a paz à terra; não vim trazer paz, mas espada;." Jesus Cristo pregou a paz ou o ódio neste mundo?

A pessoa que os evangelhos e outros livros do novo testamento chamam de Jesus, se realmente existiu, é contraditória. Leia os livros:
Porque não sou cristão - Bertrand Russell
O que Jesus disse, o que Jesus não disse - Bart Ehrman

Todos que zombam de Deus cedo ou tarde vai pagar as suas contas. Nem que seja com 100 anos de idade. Porque a posse de tranquilidade, é que algo está errado.

Não acho que seja educado zombar de ninguém. Mas não vejo como é que uma pessoa que zombe de Deus pagará suas contas, uma vez que Deus não existe. E não entendi o que você disse a respeito da posse da tranquilidade ser algo errado.

Não bebo, não fumo, não uso outros tipos de droga, não faço sexo sem camisinha, nunca briguei, não colo, nunca fiz as mer.das que todo jovem faz. Quais são as probabilidades de eu me arrepender no futuro de não ter cometido os erros que toda pessoa comum comete?

Não acho que você vai se arrepender nunca de não ter feito as besteiras que muitos fazem. Pelo contrário. Acho que você vai é ficar muito satisfeito de sempre ter agido como age.

Qual é a melhor motivação para aprender alguma coisa?

Estar completamente fascinado, fissurado, maravilhado com o assunto. Querer saber tudo a respeito, não importa o quão difícil e trabalhoso seja. Quem não tiver esse espírito a respeito do que pretenda aprender, não vai dar o máximo de si para esse aprendizado, que pode envolver grandes sacrifícios.

Acredita que a luta armada durante a ditadura militar foi algo válido? Os sequestros, roubos, etc, que ocorreram na época por parte destes grupos, pode ser, de alguma forma, justificáveis?

De modo nenhum. Acho que é válido pegar em armas para derrubar um governo despótico. Mas não é válido levar sofrimento a pessoas inocentes. Quem atua como mocinho não tem o direito de se valer das mesmas armas covardes que os bandidos se valem.

Eu, com 15 anos aguentaria um curso de cálculo caso estudasse os pré-requisitos? Uma pergunta, o cálculo é usado para que no exercício da profissão de físico, engenheiro e matemático?

Aguentaria. Só que o cálculo depende de se saber muita coisa de matemática que é dada no Ensino Médio: funções, logaritmos, função exponencial, trigonometria, polinômios e um excelente traquejo em manipulações algébricas (isso é essencial e incontornável). Naturalmente uma pessoa de 15 anos já pode estar completamente por dentro disso tudo e já saber toda a matemática do Ensino Médio ainda no Ensino Fundamental, se estudou por conta própria. Há gente assim e não são poucas pessoas não. O cálculo é a parte da matemática que estuda como as funções variam e como se acumulam. Isso é essencial no modelamento descritivo dos fenômenos naturais e sociais que se apresentam nas diversas ciências puras e aplicadas.

A evolução biológica já foi comprovada?

Claro que sim. Não há dúvidas a respeito. Isso é atestado por muitos argumentos. Muitas pessoas acham que não por não observarem ela acontecendo a olhos vistos. Mas ela acontece em períodos de tempo muito extensos, da ordem de milhões de anos. Todavia, para organismos que possuem um ciclo de reprodução rápido, como as bactérias, ela é observada a olhos vistos mesmo. Veja isto:

"Os homens são metafisicamente superiores aos animais não-humanos, […] e justamente pelo fato do homem estar acima dos animais que ele pode ter responsabilidades de ordem ética com outras espécies". Ernesto, pode comentar sobre essa resposta?

A superioridade humana em relação aos outros animais existentes na atualidade neste planeta (mas não necessariamente existentes em outros planetas ou, mesmo neste, em outras épocas do passado ou do futuro) não é de ordem metafísica e sim de ordem biológica, em especial, neurológica. É apenas a complexidade bem maior de seu sistema nervoso, especialmente do cérebro, que faz o homem possuir capacidades não apresentadas por outros animais no mesmo grau elevado. Mas outros animais também exibem algumas capacidades que os humanos exibem em grau reduzido, inclusive o senso moral. No caso humano, essa capacidade existe em grau bem pronunciado. Mas pode ser que alguma outra espécie já extinta (como a neandertal) ou a surgir pela evolução, ou ainda existente em outro planeta, a possua em grau mais elevado ainda. Essa capacidade de elaborar juízos éticos é que, certamente, faz os humanos apresentarem responsabilidade para com as outras espécies mesmo. Bem como o fato de apresentarem os sentimentos de compaixão, de empatia, de solidariedade, de generosidade, de altruísmo e outros correlatos. Isso, contudo, não é metafísico. A espécie humana não pertence a outra categoria ontológica das demais espécies animais. Tudo o que os humanos apresentam de diferente é, meramente, uma questão de grau e não de natureza. Seria de natureza, se os humanos possuíssem a propalada "alma", que os demais animais não possuem. Mas como alma não existe, os humanos, ontologicamente, estão, simplesmente, na categoria de animais.

A respeito da disponibilização de material por parte do professor eu ouvi que ele só tem a obrigação de dar a aula material o aluno que se vire... isso é correto?

Não é correto. Se ele faz uso de algum recurso além de textos em livros e apostilas, como transparências ou apresentações de slides ou em programas digitais, ele tem a obrigação ética de disponibilizar cópias disso para os alunos, mesmo que não seja obrigado. Se não o fizer é uma pessoa de mau caráter.

Professor você pratica meditação ou alguma técnica relaxante? Você usa alguma técnica para melhorar a sua capacidade de concentração nos estudos?

Não. Nem uma coisa nem outra. A única coisa que faço para relaxar é dormir à noite. E não faço uso de técnica nenhuma de aprendizagem, exceto meter a cara no estudo pra valer. Minha concentração é espontânea. Quando mergulho para estudar algo, desligo-me completamente do resto do mundo sem fazer esforço nenhum para isso. Acontece naturalmente. Sempre fui assim, desde criança.

Você acha que o bem das religiões se sobressaem sobre o mal que elas causam?

Não. Acho que o mal que elas provocam só elas são capazes, mas o bem que fazem pode ser feito sem elas. Portanto elas não fazem falta nenhuma e seria muito melhor que não existissem pois, então, nenhuma mal referente a elas seria feito. O grande mal é a enganação que elas promovem ao apresentar inverdades como verdades, como a existência de Deus, da alma humana, do céu, do inferno, de anjos, do poder da oração e muito mais. Isso faz as pessoas se conformarem com as injustiças, o que é péssimo. Além de levar muitos a confiar em meios sobrenaturais para a solução dos problemas da vida, como os de saúde, financeiros, amorosos e vários outros, em vez de tomarem providências práticas para resolvê-los.

Professor o senhor acha que a democracia corre risco nesse momento?

Não. O processo de impedimento da presidente, mesmo que eu ache que não haja razão para tal, está correndo completamente dentro da legalidade e normalidade democrática.

Ernesto, no livro Curso de Física de Berkeley, vol 1 há a informação de que a curvatura do espeço é da ordem de 10^17 cm. Isso ainda é válido e, o mais importante, o que quer dizer isso?

Atualmente, as medidas mais acuradas, conduzem a um valor nulo para a curvatura do espaço. Esse número se refere ao inverso do raio de uma esfera que contenha, como círculos máximos, as geodésicas, isto é, as linhas sobre as quais se tem as menores distâncias entre dois pontos. O número que você apresentou não é a curvatura e sim o raio dessa esfera, isto é, o inverso da curvatura.

Ernesto, entrando na licenciatura em matemática, posso trabalhar dando aulas ainda no meio do curso ou só depois de formar?

Pode lecionar como aluno sim. Desde que não haja licenciado pleiteando o mesmo lugar.

Quantas faculdades o senhor fez? Tem algum doutorado? Não que isso tenha alguma importância. É só por curiosidade. O senhor discorre praticamente sobre qualquer coisa. Considera-se um Polimata?

Sou licenciado em Matemática e mestre em Física, além de ter feito muitas matérias dos bacharelados de Matemática e Física. Veja em minha página inicial o meu perfil que está em "Sobre Ernesto von Rückert". Não discorro sobre qualquer coisa. Discorro sobre Física, Matemática, Engelharia, Filosofia, Educação, Cosmologia, Religião e um pouco sobre Evolução, Psicologia e Neurociências, Filosofia Política, História, Geografia, Música, Artes Plásticas, Literatura, Linguística, Informática e mais alguns assuntos. Mas não entendo nada de Administração, Direito, Negócios, Finanças, Agricultura, Esportes, Moda, Turismo e muitos outros assuntos. Se sou polímata ou não, os outros é que têm que dizer.

Estamos enfrentando a fase mais aguda de nossa crise política.. Neste momento, qual é o aconselhamento para seus seguidores?

Que não permitam que o impedimento da Dilma (caso ocorra) interrompa o processo de limpeza dos corruptos na política e no empresariado, sejam de que facção forem. Que lutem para a faxina completa da corrupção em todos os níveis da administração pública e das atividades privadas. Que a Lava Jato seja uma ação continuada por décadas a fio, até que todo o país se livre de qualquer resquício de corrupção e que a virtude seja a norma em todas as atividades.

Os humanos são superiores aos animais não-humanos? Vi um homem falando sobre isso e justificando o pq do veganismo ser balela, q sem dúvida os humanos são superiores pq conseguimos nos desenvolver, temos raciocínio lóg e fazemos a civilização progredi+ diferente de animais que só se reproduzem e morrem

Sem dúvida, a espécie humana, dentre os animais atualmente existentes neste planeta (mas não necessariamente em outros momentos e em outros lugares), apresenta características distintivas que a colocam em um patamar mais elevado de capacidades cognitivas. Isso não significa que as demais também não as possuam em graus variados, inferiores aos da humana. A questão ética do vegetarianismo e do veganismo é que elas também apresentam senciência, isto é, são capazes de sentir prazer e sofrimento. O que se põe é o seguinte: não é ético querer para outros o que não se deseja para si mesmo. No caso, o sofrimento. Desejar o sofrimento para si é uma patologia psíquica. Uma pessoa humana saudável não o deseja. Logo não pode desejar para outrem. E esse outrem inclui qualquer ser capaz de experimentar sofrimento. Logo matar um animal senciente não é ético. Quanto ao raciocínio, os animais também o possuem em maior ou menor grau. Eles refletem sobre suas escolhas e tomam decisões. Essa atitude especista, de considerar a espécie humana um caso especial (talvez por supor que possua alma e as demais não) é similar a uma espécie de racismo, que considera que alguma raça da espécie humana seja superior a alguma outra. Mesmo que haja alguma diferença em algum aspecto, isso não significa superioridade em termos de valor. Da mesma forma, todas as espécies sencientes são equivalentes eticamente no que tange ao critério de não ser ético qualquer ação que provoque dor, sofrimento, prejuízo, tristeza ou o que seja de ruim, intencionalmente, a outrem.

Professor, se por algum motivo vc tivesse que começar sua faculdade com 25 anos, vc iria perseguir seus sonhos ou iria se achar velho? Outra: até que ponto devemos nos importar com o que os outros vão pensar, julgar a nosso respeito? Mesmo que essas pessoas sejam especiais?

Eu começaria a fazer minha faculdade com qualquer idade, se esse fosse o meu sonho e se eu só tivesse oportunidade de concretizá-lo mais velho. Mesmo que já tivesse setenta anos. Mas, nem sempre se fazer uma faculdade é algo importante na vida. Todavia, se for, nunca é tarde para começar. Quanto ao que os outros pensam, há dois casos. Em relação às pessoas que não importam para nós, não se pode dar ouvido nenhum ao que elas pensam. Digam o que quiserem. Quanto às pessoas importantes para nós, e preciso convencê-las de que estamos com a razão. Se elas não se convencerem, então é preciso que tomemos a atitude que as contrariam, pois a vida e nossa e não delas. Se elas nos querem bem, vão aceitar. Se não aceitarem é porque querem mais o bem delas do que o nosso e, talvez, não mereçam o apreço e afeto que temos por elas.

Voltando à pergunta do PowerPoint: e se caso o professor não disponibiliza? Como proceder? Ao menos na minha faculdade, existem no mínimo uns 5 que metralham informações enquanto os slides estão rolando, e não disponibilizam os mesmos.

É só reclamar e pedir para disponibilizar. Se não for atendido, reclame na coordenação do curso, na chefia do departamento, na diretoria da escola ou onde for que resolva. O professor tem a obrigação de disponibilizar. Não tenha medo de retaliações. Nenhum professor pode retaliar um aluno se ele resolve corretamente as questões de avaliação que são propostas. Mas, para reclamar, tem que ser bom de serviço.

Olá Prof. Ernesto, o senhor poderia me indicar certos problemas (experimentais e teóricos) na qual a Relatividade não consegue explicar? Como por exemplo a viagem no tempo.

Viagem no tempo é algo que a relatividade consegue explicar. Para começar, estamos sempre viajando no tempo. A questão é se pretender avançar no tempo mais rapidamente do que os outros. Isso é possível desde que se movimente, em relação aos outros, com grande velocidade. A questão é obter essa velocidade. Ir ao passado a relatividade mostra que não é possível.

Professor Ernesto, uma pessoa que consegue entender a filosofia e o raciocínio dos filósofos, metafísica, lógica e retórica, consegue aprender exatas? Detesto resolver exercícios ou copiar no caderno o assunto que o professor passa, simplesmente, não vejo sentido em copiar.. Ainda tenho solução?

Não precisa copiar o que o professor passa. Tem tudo no livro. É preferível prestar atenção e só anotar as observações e o que ele diz que não tem no livro. Mas, para isso, você tem que já ir para a aula tendo lido as partes do livro que serão abordadas. Mais importante do que fazer exercícios e ter entendido e compreendido muito bem a teoria e os conceitos, bem como as deduções. Mas é bom fazer alguns exercícios e, principalmente os problemas. Sem esquecer de responder aos questionários. Eles são mais importantes do que os exercícios.

Mechanics do Sakurai?

Os do Landau são muito bons. Estudei neles (em Francês) e gostei muito. Não conheço o do Sakurai.

Professor me desculpe, errei na pergunta. Quis dizer que tais veículos de comunicação(Veja, Globo, Carta Capital etc.) são de fato considerado parciais. Mas tendo em vista seu conselho pergunto, você considera algum veículo da mídia realmente imparcial? Ou isso é uma utopia?

Nenhum é imparcial. Tem que ler vários para chegar à própria conclusão.

Qual a sua opinião sobre a Igreja da Eutanásia?

Completamente maluca.

Por que dois planetas não podem orbitar uma estrela na mesma linha?

Podem, desde que eles estejam localizados nos "pontos de Lagrange" da órbita, que são pontos separados por ângulos múltiplos de 60°. Fora desses pontos eles ficam instáveis e se deslocam para uma situação de colisão. Veja isto:
https://pt.wikipedia.org/wiki/Pontos_de_Lagrange
https://en.wikipedia.org/wiki/Lagrangian_point

Como estudar do jeito certo para provas de exatas?

Pegar o livro que expõe o assunto e ir lendo e acompanhando o raciocínio em um caderno de apontamentos para ser capaz de proceder às deduções por conta própria. Anotar as definições com comentários sobre as situações em que se aplicam e não se aplicam, fazendo o mesmo para as leis empíricas. Isso é importantíssimo. Há muita gente que erra muito ao aplicar alguma lei em situação em que ela não se aplica. O mesmo vale para qualquer fórmula. Tem que ter um caderno para ir desenvolvendo uma espécie de apostila pessoal com comentários, pela qual, depois, se vai estudar. Não é preciso fazer muitos exercícios, mas tem que fazer alguns. Também é preciso resolver problemas (problemas não são exercícios, são situações complexas que precisam ser resolvidas com a aplicação de várias fórmulas e a dedução de outras ainda não apresentadas). O importante, também, é responder questionários. Ache um livro que tenha questionários. Os conhecimentos conceituais são extremamente importantes. Os livros do Halliday, do Sears-Zemansky e do Knight têm questionários.

Prof, como proceder anotações numa aula ministrada por power point? se eu anotar os slides perco a explicação do professor se eu presto atenção ao professor não copio nada como resolver??

Simples. Fique prestando atenção e peça ao professor para disponibilizar os arquivos em Power Point pela internet.

Quais são as áreas mais interessantes da física? Eu me interesso pela física de partículas, astrofísica, cosmologia e física nuclear.

Isso depende do gosto de cada pessoa. Eu também acho essas as mais interessantes. Mas há quem ache outras e também está com a razão.

Comente: "Caso Deus exista, sua não-onisciência se justifica pelo fato de que, se onisciente e tudo observando, faria com que todas as funções de onda do Universos estivessem colapsadas."

Para começar, Deus não seria uma entidade física. Portanto, o conhecimento que ele teria do mundo não seria por absorção de ondas eletromagnéticas e, portanto, não interferiria no colapso de função de onda de sistema nenhum. As pessoas que consideram que Deus exista (ou que qualquer espírito exista), não fornecem explicação nenhuma sobre o mecanismo pelo qual tais entidades teriam alguma interação com a realidade física do mundo.

Apostaria no Bolsonaro pra presidente?

De modo nenhum. Discordo de quase tudo o que ele concorda.

Poderiam 2 planetas orbitarem uma estrela no mesmo "lugar" só que um embaixo do outro?(Exemplo na imagem) http://prntscr.com/at8xid

Não. O plano de suas órbitas teria que passar pelo centro de estrela e só o azul e que é assim. O verde teria que ter uma órbita inclinada em relação ao azul. Como eles estariam à mesma distântcia, teriam o mesmo período de revolução. Se foram formados em lados opostos da estrela, jamais se encontrariam. Mas se foram formados em posições não opostas, a atração mútua deles os fariam se aproximar gradativamente até se chocar.

Voce acha que cientistas também deveriam estudar filosofia? Se sim, quais filosofias (da ciencia,politica,social,metafisica)?

Certamente que sim. Especialmente lógica, epistemologia, metafísica e ética. Mas é bom estudar o resto também. E é bom estudar retórica e dialética (artes de comunicar e argumentar).

Olá Professor. O que você acha da do papel desempenhado pela mídia brasileira(especialmente nesses últimos meses)? Qual sua opinião sobre veículos considerados imparciais como Globo, Veja, Carta Capital etc.? E qual seu conselho para melhor nos informarmos nessas épocas difíceis? Obrigado.

Esses veículos não são imparciais. Nenhum o é. O que resolve é se informar a partir de vários deles e tirar suas próprias conclusões. É bom ler, também, a Época, a Isto É, o Estadão, a Folha de São Paulo e ver os noticiários televisivos da Globo, da Band, da Record e do SBT. Como não dá para comprar tudo, o jeito e ir a uma biblioteca. Felizmente trabalho em um colégio cuja biblioteca tem assinatura de vários desses.

O que você acha do golpe político contra a democracia?

Acho que não há golpe nenhum. Considero que não haja razão para que a presidente Dilma seja impedida de governar, mas acho que o processo para que isso aconteça (ou não) está sendo conduzido de forma correta. Certamente que ele envolve não apenas questões de ordem jurídica, mas de ordem política também. Todavia não se está aplicando golpe nenhum. Está se agindo dentro da lei. Para mim não há razão para impedimento. Mas eu não estou nada satisfeito com o governo Dilma e acho que o Brasil estaria melhor sem ele. Não acho que nenhum benefício que os governos petistas concederam ao povo seja justificativa para perdoá-los pela corrupção que aconteceu. Como não acho que a queda da Dilma implicaria na extinção do Bolsa Família e do Minha Casa, Minha Vida. E não acho que o fato de não se estar (ainda) julgando e condenando a corrupção de outros governantes justifique se abster de condenar os que agora estão sendo julgados. Os outros também terão que ser submetidos ao mesmo processo. Acho que se a Dilma não for impedida, seu governo ficará inviável. Também acho que sua substituição pelo Temer não será uma solução reparadora. Pelo Cunha ou pelo Calheiros será pior ainda. Novas eleições seriam ideais. Mas Aécio, Serra, Marina ou outros que estão aí também não servem. Preferiria o Cristóvam Buarque. Ou, talvez, o Ciro Gomes, mas não sei. Estamos num buraco mesmo, não importa o que venha a acontecer.

Com a crise política e econômica que se instalaram no país, a cidadania também se torna instável?

Não vejo que fique. Mesmo com esse caos político e econômico, as instituições básicas continuam vigentes, como a justiça, a democracia, a administração pública e o resto de tudo o que constitui a estrutura da nação, o que inclui as noções de cidadania, de liberdade e de responsabilidade. Nada disso está suspenso.

Qual a diferença entra Didática e Metodologia ?

Metodologia é algo genérico, aplicável a qualquer procedimento. Didática é o caso particular da metodologia aplicada ao processo de ensino e aprendizagem.

Quais argumentos para convencer um prof de química de que esta ciência é simplesmente uma parte, grande, da física ?

Mas isso é óbvio. Inicialmente as duas ciências tiveram origens distintas, a Física se ocupando dos fenômenos concernentes aos movimentos e às interações e a Química se ocupando dos fenômenos concernentes à composição da matéria e suas transformações. Com o advento da Física Quântica, contudo, viu-se que os fenômenos químicos são decorrentes de interações e movimentos dos elétrons nos átomos, de modo que a Química passou a ser um capítulo da Física. Todavia, pela tradição histórica e pelo grande volume de assuntos que ela abrange, continuou a ser considerada uma ciência à parte. Do mesmo modo que a Biologia pode ser entendida como um capítulo da Química, mas é considerada uma ciência à parte, dada a complexidade dos fenômenos envolvidos.

http://www.paulopes.com.br/2016/04/filosofia-eh--linha-morta-de-pensamento-diz-hawking.html#.VxK_NvkrLZ4 O que pensa a respeito?

Discordo completamente de Hawking. Há muito tema que a Filosofia aborda e que as ciências não o fazem. Claro que muito do que antes era coberto pela Filosofia, hoje o é pelas ciências, como a origem do Universo e a origem da vida. Mesmo assim a Filosofia tem o seu lugar garantido, pois ela é que sistematiza, critica e dá valor ao conhecimento, mesmo científico. E que estabelece o método de se buscar o conhecimento. Além de outros aspectos, como a ética e a estética. A própria Metafísica, todavia, tem o seu lugar muito bem posto no concerto dos conhecimentos. E não estou me referindo, absolutamente, a qualquer cogitação a respeito de realidades sobrenaturais. Mas a metafísica promove a categorização da realidade, o estabelecimento de condições de existência e muitos outros aspectos. Para mim, proclamar a morte da Filosofia é demonstração de ignorância.

Energia é a capacidade de realizar trabalho. Trabalho é a capacidade de transferir energia. É isso ?

Não. Energia é a propriedade dos sistemas que os capacita a agir, ou realizando um trabalho, ou transferindo calor, ou emitindo radiação, por exemplo. Trabalho é uma das modalidades de transferência de energia entre sistemas, ao lado do calor e da radiação.

Por que não gosta da Psicologia Clínica?

Porque é algo que não me cativa, não me prende. Não acho interessante. Além do mais, considero que se trata de um conhecimento muito doxista, pela existência de várias escolas de concepções distintas sobre as origens e razões dos problemas psíquicos e sobre os métodos a se usar para seu tratamento. Não vejo solidez epistêmica nisso.

Professor, qual a diferença entre trabalho e momento? Ambos são calculados pelo produto da força x deslocamento não é?

A diferença é que o trabalho é o produto do deslocamento (isso é, distância percorrida) pela componente da força na direção do deslocamento, sendo uma grandeza escalar. Por sua vez, o torque, ou momento da força, é o produto da força pela distância (e não deslocamento), da linha de ação da força até o ponto em relação ao qual o torque esteja sendo calculado, tratando-se de uma grandeza vetorial de direção perpendicular ao plano da força e dessa distância e sentido dado pela "regra da mão direita".

Gosta da psicologia?

Sim, mas não do aspecto terapêutico. Gosto da psicologia como um estudo do funcionamento e da estrutura da mente e de todos os seus processos e propriedades, bem como de sua natureza. Tudo em estrita correspondência com a neurologia. Para mim a Psicologia é uma divisão da Biologia. O psiquismo é, em verdade, função da anatomia e da fisiologia do cérebro, seus anexos, todo o sistema nervoso, os órgãos dos sentidos, as glândula endócrinas e, enfim, todo o organismo. Não existe componente nenhum de natureza não biológica na mente, ou seja, não existe alma espiritual. Assim concebida, considero a psicologia, juntamente com a neurociência, um assunto extremamente interessante de estudo, justamente para entender a inteligência, o comportamento, as emoções, os sentimentos, a vontade e tudo o mais que faz parte da vida psíquica.

Nas palavras do grande físico Max Plank: "real é tudo aquilo que é mensurável"(em termos da física-matemática. Como seria a natureza de outra forma? Poderia dar-nos um exemplo?

Real é tudo o que existe independentemente de alguma mente, mesmo que não seja mensurável. Em suma, real é o que não é abstrato, isto é, não é puramente conceitual. É o que é objetivo, independente de um sujeito. Mas real não são apenas seres. Relações, atributos, propriedades, ações, ocorrências, estruturas, configurações, estados e outras coisas também são reais. O que pode ser mensurável são propriedades de seres ou de outras categorias, dos tipos que mencionei. São extensões, intensidades, quantidades, durações. Não estão me ocorrendo exemplos no momento, mas há propriedades ou categorias reais, objetivas e não mensuráveis. Vou procurar e passar. Não acho que o fato de ser mensurável seja uma condição fundamental para que algo seja real. A descrição da realidade em termos das propriedades que a humanidade, historicamente atribuiu a cada coisa tem a ver com o modo pelo qual os órgãos de sentidos humanos a percebem. Outros seres, dotados de outros tipos de sentidos, percebem a realidade de modo completamente diferente, como cães, morcegos, alguns peixes que sentem campos elétricos, insetos e outros tipos de animais. Seres extra-terrestres, então, poderiam ter percepções completamente distintas. Isso os poderia levar a construir modelos descritivos, que suas mentes assimilariam, com base em conceitos completamente diferentes e a ciência deles não possuir conceitos como força ou energia e, mesmo assim, ser capaz de construir modelos que conduzam a previsões que confiram com o que acontece. Esses conceitos que exemplifiquei, e todos os outros, não existem na natureza em si, mas no modo como a humanidade elaborou seus modelos descritivos dela.

Opinião,sobre um filho crescer com pais separados? Pode ser prejudicial a criança?

Não necessariamente. Depende de como esses pais se entendem entre si e concebem o modo de interagir com o filho. O fato de serem separados não significa que sejam adversários ou inimigos. Eles podem e devem ser amigos, mesmo que se envolvam com outros relacionamentos. E os parceiros desses novos relacionamentos têm que aceitar o fato de que eles são pais comuns de uma mesma criança e que isso implicará em que tenham uma relação a ser mantida. Os novos parceiros precisam ser amigos do parceiro anterior e, também, dedicarem ao enteado o amor paternal ou maternal que dedicariam aos próprios filhos.

Professor, fazia eng. mecânica, fiz 3 semestres e mudei pra elétrica em outra cidade e decidi n aproveitar nenhuma matéria. Estou no segundo semestre mas acabei gostando tanto de cálculo, álgebra, etc que acho que queria matemática. Mas estou com 21 anos, você acha que devo mudar mesmo assim?

21 anos ainda é estar novo. Pode mudar, se for isso que vai te deixar realizado na vida. O importante é se fazer o que gosta, o que vai dar satisfação, aquilo em que se pensa que se pode dar uma boa contribuição para o bem do mundo. Sugiro que faça a licenciatura e o bacharelado e, depois, o mestrado e o doutorado. Enquanto isso, com a licenciatura, já pode ir ganhando dinheiro com o magistério.

A matemática fornece a linguagem na qual as ciências naturais aspiram descrever e analisar o universo. No entanto, muitas pessoas classificam a matemática como uma "ciência exata". Afinal, matemática é ciência ou uma espécie de linguagem da natureza?

Nem uma nem outra. A matemática é uma linguagem, mas não é da natureza. Ele foi inventada pela humanidade e se presta à construção de modelos descritivos do comportamento da natureza. Mas a natureza, em si mesma, não é matemática. Nossa interpretação da realidade é moldada pelo fato de que a analisamos matematicamente. Mas poderia ser de outra forma.

Professor, e a ideia de fazer vídeos para o YouTube (como o Clarion, Pirula...)? Desistiu?

Não. Mas como meu trabalho me absorve de dez a doze horas por dia (e, muitas vezes, fins de semana) e eu ainda não aprendi a passar a vida sem dormir, não acho tempo para isso. É que também tenho outros interesses pessoais, como ler e estudar. Já não vejo televisão e durmo pouco, mas, mesmo assim, o tempo não dá.

A inteligência linguística se refere à habilidade de se comunicar o à aprendizagem de idiomas?

Sim, bem como a entender e interpretar textos. A comunicação envolve a parte oral e a escrita. Quem tem boa inteligência linguística faz discursos de improviso, escreve livros com facilidade, aprende outras línguas facilmente, entende o que escuta e o que lê com facilidade também. Isso, é claro, envolve o conhecimento de vasto vocabulário e o entendimento e traquejo de uso das regras gramaticais. Mas esse conhecimento, entendimento e traquejo são possibilitados facilmente, justamente, pela inteligência linguística.

Como calcular o perímetro de uma elipse?

Integrando o elemento de linha. O melhor é expressar em coordenadas polares. Comece pela equação x²/a² + y²/b² = 1 e faça x = rcost e y = rsent. O elemento de linha é dado por ds² = dx² + dy². Expresse tudo em termos de t e integre de t = 0 a t = 2pi.

Ernesto,recentemente um relacionamento meu acabou,devido ao fato de eu querer retomar a vida de estudante...A pessoa não aceitou,e na visão dela eu que estou errado! O que acha? rs

As pessoas têm concepções diferentes sobre a vida. Em se tratando de sua própria vida, as decisões têm que ser tomadas em função de suas próprias concepções. Claro que é válido tomar opinião de outras pessoas, mas quem tem que resolver é a própria pessoa, inclusive porque, se der errado, a responsabilidade sempre tem que ser da própria pessoa, que não pode por a culpa em ninguém, porque, mesmo que ela tenha acatado a opinião de outra pessoa, ela a aceitou. Então a outra pessoa não tem responsabilidade por fracasso nenhum e nem pelo sucesso. Como não sei quais sejam as condições de sua vida, não posso opinar se você ou a pessoa com quem você se relacionava é que estava certa. Mas, mesmo que você esteja errado, você tem que resolver sua vida em função do que você mesmo acha. E assumir todas as consequências, sem reclamar.

Ernesto, segundo alguns estudiosos a lei da atração pode ser comprovada através da mecânica quântica. Você sabe se é realmente verdade isso?

Nada disso. Isso é conversa fiada de gente que não entende patavina de Física Quântica e fica inventando conexões esotéricas com ela. Não tem o menor fundamento.

Você falou em conservação do momentum no surgimento de partículas a partir do campo do vácuo e dos fótons resultantes dos aniquilamentos. Mas como isso acontece? O campo não possui momentum certo? Então como o mesmo pode ser conservado?

Campos possuem momentum sim. No caso dos campos elétrico e magnético, veja este artigo:
http://profs.if.uff.br/moriconi/cederj/aula14.pdf
No caso do campo gravitacional, veja este artigo:
http://www.cbpf.br/EscolaCosmo/pmc_Vitorio.pdf
(o autor deste último foi meu aluno do Bacharelado em Física e fez iniciação científica comigo sobre Relatividade Geral).
No caso do campo da matéria, não sei a matemática correspondente, mas a lei de conservação do momentum, do momento angular, da energia e outras se aplica.

(Pergunta pessoal) Ernesto, você já foi casado algumas vezes. Por que esses relacionamentos tiveram fim?

Só um relacionamento conjugal meu é que teve fim. Estou no segundo. O primeiro acabou em razão de divergências de cosmovisões e perspectivas de vida, isto é, do que cada um deseja construir na vida, que projetos possui, que significado atribui à própria vida. É importante que tais considerações sejam convergentes para que pessoas que se unem para constituir um núcleo familiar se sintam realizadas por participarem dele e que o trabalho conjunto contribua para a realização pessoal dos envolvidos. Se o que uns querem da vida seja conflitante com o que outros querem da vida, a condução da convivência não fica harmônica e não conduz a que cada um contribua para a satisfação de viver dos demais.

Como se desenvolve o gosto pelos estudos? Não gostar de estudar, descaracteriza a inteligência de uma pessoa?

A pessoa pode não gostar de estudar e ter inteligência voltada para outros aspectos. Como também pode não ter inteligência (isto é, ter baixa inteligência). Os tipos de inteligência que se revelam pelo gosto de estudar são as inteligências lógico-matemática, linguística e, geralmente, espacial. As inteligências cinestésica, interpessoal, intrapessoal e emocional não estão relacionadas a estudo. A musical pode estar ou não. Há, também, uma inteligência dita "global", que se revela pela grande curiosidade em querer saber tudo sobre todos os assuntos, não só conhecimentos, mas, também, domínio de habilidades. Em geral que tem essa inteligência, também gosta de estudar e frui um grande prazer com isso. A cinestésica pode estar vinculada a um gosto por estudo, quando é voltada, por exemplo, para artes plásticas. Mesmo quando é voltada para esportes, pode envolver o gosto pelo estudo do esporte.

O que acontece quando uma partícula e uma antipartícula se colidem? Elas viram o que?

Elas se transformam em dois fótons, cujas energias somadas correspondem às massas e as energias somadas delas, pela relação E = mc². É preciso que sejam dois fótons e não um só, para garantir a conservação do momento linear. Intermediariamente, entre o desaparecimento da partícula e sua antipartícula e o surgimento dos dois fótons, há um estado de "campo do vácuo puro", que o César Lattes chamou de "bola de fogo", em que as massas e as energias cinéticas e potenciais das partículas ficam na forma de energia do campo, até se tornarem energia dos fótons. É o que muitos denominam "energia pura", mas isso não existe. Energia não pode ser algo puro. Tem que ser a propriedade de alguma coisa.

Como lhe dar com dor de cotovelo? Quando se é dispensado por alguém do qual você gosta, como sair da ''fossa''???

Para "lidar" com a "dor de cotovelo" é preciso se ter uma visão dos relacionamentos humanos bem abrangente, isto é, considerar que não existe uma única pessoa no mundo que preencha nossos anseios amorosos. Se se encontrou uma e o relacionamento não se estabeleceu, certamente que isso é doloroso. Mas é preciso considerar que muitas outras também existem, que sejam capazes de preencher os mesmos anseios, só que não foram encontradas. Há casos, inclusive, de se encontrar mais de uma outra e se ficar completamente aturdido por se ter que fazer uma escolha. O ideal é que essa escolha não precisasse ser feita e se pudesse ter relacionamentos plurais inteiramente consentidos pelos envolvidos. Pode ficar certo de que haverá outra ou outras pessoas que suprirão as carências que a que não correspondeu supria.

Professor, estudando mecânica quântica muitas vezes nos deparamos com o famigerado "observador". Sem o uso de analogias, o observador é uma entidade/abstração matemática ou é realmente uma "pessoa" que provoca o colapso da função de onda?

Não é uma pessoa, necessariamente. O que provoca o colapso da função de onda é o fato dela enviar alguma informação sobre si, mesmo que essa informação não seja captada por nada.

Ernesto, qual sua opinião sobre o Anarco-primitivismo? há algo que possa ser aproveitado dessa ideologia?

Não acho que seja interessante. Acho que o progresso científico, cultural e tecnológico são bens de grande valor que devem ser cultivados e fruídos dentro de um esquema político anarquista e econômico comunista. Concebo o anarco-comunismo como uma sociedade sofisticadíssima em termos de cultura, educação, artes, ciência e tecnologia.

O que você acha de quem diz : "As pessoas vivem hoje melhor que os rei da idade média graça ao capitalismo"?

Vivem, mas não é graças ao capitalismo e sim graças ao progresso científico, tecnológico e social, que também seria atingido sem o capitalismo e, se o comunismo (e não o socialismo de estado) já fosse um sistema econômico milenar, o progresso teria sido muito maior.

Conhece o método Kumon? Eu tive uma péssima formação matemática e estou frequentando para eu ter uma boa formação matemática e entrar preparado para enfrentar os cálculos pesados de Física e computação.

Conheço e acho muito bom. Mas é lento.

As flutuações quânticas do campo de dirac apenas condicionam e não determinam o surgimento de pares de partícula e antipartícula? Desse modo, o surgimento de matéria a partir do campo do vácuo é um evento incausado?

Sim, exatamente

sexta-feira, 5 de maio de 2017

Tu acha que o anarquismo daria certo hoje em dia? ou o comunismo? Li algumas respostas suas sobre esses temas, mas ainda estou confuso e procuro informações para formar a minha opinião. Já eu acho que não daria muito certo, em partes.

Não daria certo hoje, de modo nenhum. O anarco-comunismo é um objetivo a ser alcançado pela evolução civilizatória que demanda uma profunda mudança de cosmovisão, de mentalidades, de atitudes. Isso é uma reforma gradual a ser conseguida ao longo de muitas gerações o que demandará séculos.

O que acha? http://www1.folha.uol.com.br/mercado/2016/04/1760052-dinheiro-pode-sair-de-circulacao-na-suecia-ate-2030.shtml?cmpid=facefolha

Acho bem interessante. Mas o que eu quero não é a abolição do papel moeda e o uso exclusivo da moeda eletrônica. O que eu quero é a abolição da própria moeda, isto é, que tudo seja inteiramente de graça e que todos trabalhem de graça. Sem moeda de espécie alguma. E nem com economia de troca. O que eu quero é a economia de doação. Todo mundo trabalha e doa o produto do seu trabalho a troco de nada. Todo mundo fazendo isso, uns pelos outros, todos terão tudo. Ninguém será pobre e ninguém será rico.

Qual a integral indefinida da função y=(1-x^2)^0.5?

(1/2)(x(1 - x^2)^(1/2) + arcsenx)
Faça x = sen z e integre por partes.

Velocidade constante é a mesma coisa que aceleração constante?

Absolutamente não! Velocidade constante significa que se percorrem distâncias iguais em tempos iguais. Aceleração constante significa que a velocidade varia de quantidades iguais em tempos iguais.

Qual a sua opinião em relação ao preconceito linguístico?

Todo e qualquer preconceito é uma idiotice e revela um caráter mesquinho e uma mentalidade obtusa.

Trabalhas bem sob pressão?

Sim, mas não gosto nem um pouco. Gosto de fazer tudo sem pressa e sem pensar que tenho algum tempo para acabar. Porque sou muito perfeccionista e não tolero nada que fique simplesmente bom, ou mesmo ótimo, e não excelente. Só que excelência requer complexidade e complexidade requer muito trabalho e grande dispêndio de tempo. Daí, quando faço algo, vou fazendo, sem a menor preguiça, mas sem me preocupar quando vou acabar. Não penso em descansar e nem ligo para trabalhar vinte horas seguidas e dormir só quatro horas. Mas não adianta me afobar que eu não tomo conhecimento. Vou continuando a fazer o que é preciso para que tudo fique perfeito. Sou muito independente e não sou nem um pouco obediente. Sempre fui assim. Faço o que eu quero, como quero, quando quero. Mas as pessoas sempre estão me chamando para fazer o que precisam, porque eu dou conta das coisas mais difíceis e complicadas com tranquilidade. Mas sem pressa. E fica tudo muito bem feito.

Votaria em quem como presidente?

Cristóvam Buarque.

O que o professor acha do niilismo? Diante de tantos questionamentos, venho-me a simpatizar com as premissa de Niet..embora o terreno seja perigoso, em suma, acalmou algumas inquietações, p/ fazer surgir outras dúvidas. A filosofia afinal, é isso..

Niilismo é a negação de valores de toda ordem: éticos, estéticos, políticos, metafísicos e por aí vai. Para mim isso não tem cabimento. Uma coisa é considerar que existem valores equivocados que precisam ser mudados, outra é considerar que não se deve ter valor nenhum para nada. Considero que a vida boa para todos necessita que as pessoas prezem e pratiquem valores e virtudes pessoais e sociais, como justiça, honestidade, solidariedade, compaixão, diligência, tolerância, coragem, bravura, sinceridade, assertividade, bondade, amizade, amor e muitos outros, até mesmo elegância, cortesia, bons modos. Discordo de Nietzsche quanto a isso. Como discordo de valores incorretos para serem tidos como balizadores do valor pessoal de alguém, tipo a beleza, a inteligência, a riqueza, a simpatia, a fama, a cultura, a força. Claro que são valores válidos. Só que uma pessoa não é melhor do que outra por ser mais bonita, mais inteligente, mais rica, mais simática, mais famosa, mais culta, mais forte.

Ernesto, Penso que buracos negros podem não ter nenhum conteúdo ou caroço central. A origem do seu campo gravitacional pode se localizar, simetricamente dividida, apenas na totalidade da sua superfície ou horizonte de eventos. Uma casca esférica de puro campo gravitacional, sem recheio. O que achas?

Acho impossível, por violar a conservação dos números bariônico e leptônico. O conteúdo que passa a se localizar aquém do horizonte de eventos continua a possuir esses números, além da carga elétrica, do momento angular e da massa. Um campo gravitacional puro poderia, apenas, ter a energia correspondente à massa, bem como momento angular, mas não teria carga e nem números bariônicos e leptônicos, além de outros. Por isso é que é preciso que se considere que haja um caroço de altíssima densidade, constituído de um único hiper-híperon, com o valor total de todos esses números quânticos que havia no conteúdo que passou para dentro do horizonte de eventos.

Acha que vale a pena aprender uma língua planejada, como o esperanto?

Acho que não. Uma língua internacional tem que aparecer espontaneamente a partir das que já existem. Atualmente o inglês faz esse papel. Mas ele já foi exercido pelo grego, pelo latim e pelo francês.

Quais são as diferenças entre a Esquerda Reformista e a Esquerda Revolucionária?

A diferença é o modo de se alcançar os resultados. A esquerda reformista pretende obter os seus resultados dentro do processo democrático, por meio de reformas aprovadas legalmente. A esquerda revolucionária pretende obter os seus resultados por meio de um processo violento de ruptura da situação estabelecida, com a derrubada do governo estabelecido e a implantação de um governo revolucionário autocrático que promova o estabelecimento do socialismo, normalmente estatal e não comunista, com a contenção à força de qualquer oposição. Considero que o modo revolucionário de se atingir uma economia não capitalista (isto é, sem o capital concentrado e sem a existência de trabalhadores empregados) não é bom porque promove uma situação de tensão a ser controlada à força, além de implicar em grande sofrimento para toda a população, uma vez que se torna preciso o estabelecimento de um estado policial com amplas restrições à liberdade, para evitar a contra-revolução. A forma reformista, apesar de lenta, é mais garantida, exatamente por sua gradualidade, que a torna aceitável pela maioria e passível de se ter a oposição controlada adequadamente, sem violência. A Social Democracia é uma forma gradual de Esquerda Reformista, que pode chegar até ao anarco-comunismo. O que é preciso é promover a gradual pulverização do capital e extinção do emprego, com todo o trabalho sendo executado pelos próprios detentores do capital, que seria a totalidade da população economicamente ativa, isto é, todas as mulheres e homens capazes.

A teoria quântica de campos é uma das áreas mais importantes da ciência? Uma vez que praticamente tudo adveio do campo, entendê-lo a partir da ótica quântica, é importantíssimo, concorda?

Completamente. Mesmo assim é possível se desenvolver alguma área científica sem levar até a redução extrema em termos de campos, o que acontece na Biologia, na Psicologia, na Sociologia e, mesmo, em algumas ciências físicas, como Geologia e Meteorologia. Todavia, quando se quer fazer um aprofundamento dos comportamentos nessas ciências, se acaba chegando até a base, que consiste nas interações entre as partículas elementares, que é o objeto da teoria quântica de campos.

Se houver uma explicação científica e uma explicação filosófica para algo, a científica é que deve ser adotada. Procede? Se sim, isso vale para todo o caso?

Sim, a científica tem prioridade porque é devidamente testada contra qualquer refutação por meio de observações e experimentações criteriosas. A filosófica advém de reflexões calcadas em opiniões que não são submetidas ao crivo do teste factual, mas apenas ao escrutínio da razão, que, muitas vezes, não confere com o comportamento real do mundo.

Votaria no Bolsonaro? https://www.facebook.com/desobedientesempre/videos/1325911217425364/

Nem em sonhos. Considero-o o arquétipo do político que não deve existir (direitista conservador), ao lado dos corruptos (o que penso que ele não seja) e dos demagogos. Pelo menos ele é autêntico e sincero e não como os que se dizem de esquerda e só querem se locupletar. Mas não voto em nenhum dos dois tipos. Só voto em comunista verdadeiro (e não socialista estatizante) ou em social-democrata autêntico. Que sejam de ilibação à toda prova, de renomada competência e de progressistas propostas.

O que faz um jovem ser conservador?

Eu não consigo entender o que possa fazer alguém ser um conservador ou uma pessoa de direita, mesmo que eu admita o pleno direito de o ser. Considero que tradições, se forem benéficas, podem ser conservadas (e é bom o serem), mas ter uma postura de ser a favor de tudo que represente as concepções atrasadas e retrógradas só porque são relíquias do passado a serem preservadas, para mim, é um absurdo inominável. A evolução social, cultural, científica e tecnológica existe, justamente, para superar essas concepções malsãs. Isso não significa que tudo que seja moderno e atual, só por isso, seja melhor do que o que já esteja estabelecido. Mas considerar que se deva defender o que esteja estabelecido só porque já é assim há muito tempo é completamente pirado.

A química não é parte da filosofia?

Também se originou da Filosofia, como todas as ciências. É que eu me esqueci de mencionar. Atualmente as ciências não fazem mais parte da Filosofia.

Quais são os motivos pra vc não ter desistido do ask ainda?

Porque ele é um excelente veículo para que eu possa difundir minhas ideias anarquistas e ateístas, bem como para que eu possa colaborar para espancar a ignorância do mundo, especialmente a científica, mas também artística e cultural em geral. Considerando que eu coloco uma cópia de tudo que posto nele em meu Facebook, meu Twitter e um dos meus Blogs, a penetração fica maior ainda. Por outro lado, ao responder a perguntas, fico atento a que tipo de assunto é demandado pela juventude, que são o meu maior público.

A física e a matemática são derivadas da filosofia? O método científico, é, em última instância, filosófico?

Sim, bem como a Biologia, a Geologia, a Astronomia, a Cosmologia, a Psicologia e todas as ciências. No começo tudo fazia parte da Filosofia e depois foi adquirindo a sua independência. O método científico é parte da Epistemologia que é um dos capítulos da Filosofia. A Filosofia, agora, é meta-científica.

Se forças magnéticas não realizam trabalho, quem o realiza quando aproximo dois ímãs e os solto, de modo que eles se atraem/repelem?

Não é bem assim. Forças magnéticas não realizam trabalho quando aplicadas a cargas em movimento livre. Mas se as cargas estiverem ligadas a um sistema por outras forças, como nas estruturas do estado sólido, as forças magnéticas realizam trabalho sim. Mesmo em fluxo hidrodinâmico viscoso de meios ionizados, os campos magnéticos realizam trabalho.

Professor, a modelo econômico que citou como resposta à minha última pergunta, seria exatamente este? https://pt.wikipedia.org/wiki/Economia_de_oferta https://en.wikipedia.org/wiki/Gift_economy Ou este modelo possui algumas características diferentes do que você consideraria ideal?

É mais ou menos isso mesmo, com algumas diferenças fundamentais:
1. Trata-se de um sistema que abrange a totalidade das relações econômicas entre o mundo todo, ficando excluídos inteiramente a existência de moeda e o processo de escambo.
2. Tal economia se aplicaria a uma sociedade com extrema sofisticação cultural e tecnológica, e não a uma sociedade primitiva.
3. O sistema de produção e distribuição por doação não obedece a nenhuma regra que estabeleça qualquer restrição ou quota, tampouco obrigações. Toda a atividade é voluntária.
4. Todavia, o sistema tem que estabelecer sanções sociais aos que se recusem a colaborar para o bem comum e queiram apenas se aproveitar das benesses do sistema.
5. O sistema tem que coexistir com uma organização social coletivista em todos os aspectos, sem que exista propriedade particular, exceto de bens de uso estritamente pessoal, como óculos, dentaduras ou escovas de dentes. Mas até as roupas podem ser coletivizadas. E, certamente, maridos e mulheres, com a abolição da noção tradicional de família e sua substituição por uma concepção mais ampla.
6. O sistema tem que coexistir com uma organização mundial totalmente sem fronteiras.
7. Idealmente o sistema deveria, politicamente, prescindir de governos e de estados que os abriguem.
8. Certamente que tal sistema supõe uma população escrupulosamente honesta e diligente, com a abolição completa de toda preguiça e de toda preguiça.
9. O sistema precisa contemplar uma cosmovisão das pessoas inteiramente aberta, libertária, despreconceituosa e tolerante, razão pela qual não pode admitir nenhuma religiosidade fundamentalista que não aceite outras concepções.
10. Em suma, isso significa uma situação político-econômica anarco-comunista, lembrando que anarquia não é desordem, pelo contrário, é o suprassumo da ordem e que comunismo não é controle estatal da economia, pelo contrário, é a liberação da economia inteiramente do controle governamental, com a abolição completa da relação patrão-empregado, inclusive em relação ao governo.

Qual área da economia mais lhe interessa? Digamos, se você fosse cursar um bacharelado em economia... O que iria preferir estudar? Com o que gostaria de trabalhar?

Se eu fosse estudar economia me focaria nos aspectos não financeiros e monetários da economia para apresentar minha proposta de "Economia de Doação" em que não existe moeda e nem escambo mas todo mundo trabalha de graça para produzir tudo que é doado (sem troca) para o resto da população. Mas isso não significa uma sociedade primitiva. Estou me referindo a uma sociedade cultural e tecnologicamente sofisticadíssima. Tal sociedade não teria fronteiras nacionais, sendo o mundo todo uma única entidade política, de preferência sem um estado estabelecido e sem governo. Mas isso já é um aspecto político e não econômico. A economia se focaria na produção e distribuição de bens, sem equivalente monetário. Essa é a forma de economia que preconizo para o mundo e gostaria de estudar a fundo para poder fazer uma proposta bem fundamentada. Ela é que é o verdadeiro comunismo e não esse socialismo de estado que foi chamado de comunismo na União Soviética e seus satélites e imitadores. O capital não é monetário. O capital são os bens e o trabalho. Mas não existe trabalhador empregado (muito menos assalariado). Nem do estado. Todos são, ao mesmo tempo, trabalhadores e detentores do capital, que é compartilhado. Não há quem seja só detentor de capital e quem seja só trabalhador. Além do mais, a organização social que acompanha essa economia é uma organização completamente coletivizada. Não havendo nada privado. Nem mesmo mulheres e maridos. Roupas, veículos, residências, tudo é comunitário. Os transportes são coletivos, podendo haver veículos pequenos para emergências, disponíveis em estacionamentos coletivos. As residências seriam como os falanstérios. Um paraíso na Terra. O suprassumo da civilização. Todo mundo culto, todo mundo saudável, todo mundo próspero, todo mundo muito bem educado, todo mundo solidário, todo mundo gentil. Sem polícia, sem exércitos, sem advogados, sem juízes, sem prisões, sem contadores, sem bancários (e sem banqueiros). Uma beleza.

Professor Pardal, por que políticos não entendem de economia? Visto a situação econômica atual, parece que é um assunto não muito dominado pela classe política. Isso não é contraditório, já que eles são escolhidos para gerirem a coisa pública?

Se você pretende me comparar a um tipo dos quadrinhos eu estaria mais para o professor Ludovico do que para o Pardal. Políticos, a priori, não têm que entender de nada. Nem de economia, nem de administração, nem de contabilidade, nem de direito, nem de política. Muito menos de ética, de lógica, de dialética, de gramática, de retórica. Muito menos, ainda, de matemática. E nem experiência administrativa na vida privada. Mas deveriam. Para mim seria preciso haver um exame de suficiência mínima nesses assuntos para que qualquer um pudesse se candidatar a algum cargo público. E deveria haver cursos preparatórios, como existem para concursos em geral, para as pessoas se prepararem para esse exame. Deveria, também, haver algum tipo de psico teste que aferisse a honestidade da pessoa. Se a pessoa passasse nesses exames, sua candidatura estaria homologada. E nem seria preciso ser filiado a partido político nenhum. A reforma política precisa ser feita logo e acabar com essa necessidade. Bem como a do voto obrigatório, do voto de legenda, do voto proporcional, da fidelidade partidária. Instaurar o voto distrital misto e várias outras medidas. Como a cassação dos direitos políticos por improbidade ou corrupção para o resto da vida, com confisco dos bens, inclusive dos familiares, para ressarcir os prejuízos causados.

Pq Tesla não ganhou o prêmio Nobel?

Não sei dizer. Mas sei que a concessão do prêmio Nobel envolve muitas intrigas e vaidades e o fato de Tesla (associado a Westinghouse) ter sido adversário de Edison na questão da transmissão da eletricidade por corrente contínua ou alternada, e Edison ter muito poder econômico e muitas ligações com pessoas importantes pode ter sido um fator. Leia estes artigos:

Como ser uma pessoa mais querida e prestigiada socialmente?

Em minha opinião o que faz alguém ser reconhecido, admirado, prestigiado e amado pelas outras pessoas é um conjunto de características que apresenta, como empatia, simpatia, confiabilidade, extroversão, boa educação, inteligência, segurança, solidariedade, desprendimento, generosidade, desenvoltura, cultura, honestidade, sinceridade, diligência, interesse genuíno pelos outros, afabilidade, alegria, boa retórica, boa lógica, boa dialética, boa estampa, assertividade, modéstia (mas não humildade), até mesmo elegância, não só no vestuário, mas no trato, nas posturas, no modo de falar. Ausência de empáfia, de pernosticismo, de bazófia, de gabolice, de presunção, de ignorância, preconceito, de intolerância. É um conjunto de qualidades que têm que ser cultivadas, mas que, em grande parte, são inerentes e inatas. Muitas desenvolvidas na tenra infância pela educação familiar.

Hawking disse no "Universo numa casca de noz" que o relógio atômico de um avião que viaja de oeste para leste mede um tempo menor do que viaja de leste para oeste, pois a velocidade do avião soma com a velocidade de rotação da Terra. Procede?

Procede. Leia este artigo:
http://www.fflch.usp.br/df/opessoa/TR-Exp-2-Avioes.pdf

Professor, os problemas que tratamos na Fisica Classica podem ser redescritos corrigidamente dentro da Fisica Relativista? Ou esta fica mesmo para problemas envolvendo velocidade da luz e cosmologia? Ou ainda: um engenheiro civil poderia corrigir seus calculos com o uso da relatividade?

Todas as questões da Física Clássica e da Engenharia podem ser abordados sob o aspecto relativístico. Mas essa abordagem é extremamente mais complicada e levará, dentro da precisão das medidas, aos mesmos resultados da abordagem clássica. Apenas quando são envolvidas velocidades muito grandes (acima de um décimo da velocidade da luz), ou campos gravitacionais muito intensos (milhares de vezes mais fortes que o terrestre) é que a abordagem relativística é que conduzirá aos resultados corretos e a clássica não. Portanto, não compensa o esforço de usar a relatividade onde ela não leva a resultados que difiram dos resultados clássicos. Isso se aplica, inclusive, a mandar uma nave da Terra a uma lua de Saturno. Mas a relatividade já é necessária no projeto de um acelerador de partículas e, mesmo, de um programa de posicionamento global (GPS).

Professor, estou no segundo ano do Ensino Médio e não estou achando o conteúdo da escola interessante. Ainda gosto bastante de ciência, filosofia e conhecimentos em geral. Sempre estudo coisas novas em casa. Sou burro por não gostar do conteúdo escolar?

Claro que, por isso, não é burro (mas até pode ser, por outras razões). Infelizmente os conteúdos do Ensino Médio não são passados de modo atrativo por deficiência dos professores. Ou porque, de fato, para uns e outros, tal ou qual assunto seja chato mesmo. Eu mesmo, que sempre adorei estudar, preferia estudar assuntos que não eram os pertinentes ao conteúdo escolar. Mas eu dava conta, facilmente, do conteúdo escolar por meio de dois recursos: Me aplicar intensamente em entender e compreender o que estava sendo passado nas aulas nas próprias aulas, não deixando quase nada para ser estudado depois, em casa. Para isso tem que ficar extremamente atento à aula e perguntar ao professor qualquer coisa que não entenda, não importando nem um pouquinho em pagar mico e ser chato. E nem ligando a mínima para a opinião dos outros alunos e do professor. Segundo, se ainda ficar algo sem entender, ter uma força de vontade muito grande e estudar em casa, NO MESMO DIA DA AULA, até estar tudo entendido, anotando o que não ficou entendido para perguntar ao professor NA PRIMEIRA AULA QUE TIVER. Assim você se livra do assunto, mesmo que seja chato (para isso a necessidade da força de vontade) e pode dedicar o seu tempo a estudar o que achar interessante e que não faz parte do currículo escolar. Um detalhe: preguiça não pode existir em sua vida.

Algumas pessoas me acham chato e fútil, mas o que importa é a sua opinião! O que você acha?

As respostas que estão em seu perfil realmente são superficiais. Talvez porque ninguém te pergunte nada profundo. Gostaria de ver respostas sobre questões mais significativas, para ter uma avaliação. Aguarde que eu as farei a você. Quanto às perguntas que você faz, acho pertinentes em sua maioria. Mas algumas são impertinentes.

Professor, ainda sobre o Planeta X, as variantes na orbita de Netuno não poderiam ser explicadas pela massa total do Cinturão de Kupier

Se o cinturão de Kuiper tiver uma distribuição uniforme de cometas e asteroides, não. Para causar perturbação seria preciso haver alguma região com mais concentração de massa ou, realmente, um planeta. Mas, como se detectou perturbações na órbita de Netuno devido a Plutão, que é pequeno, e ele foi achado e como não se achou nenhum nono planeta, provavelmente não seja um planeta ou planeta anão.

https://www.youtube.com/watch?v=IP98FLS50Mk&feature=share O que você acha dos argumentos dele? Não acha que usou muitos argumentos ad hominem? Concordo com algumas coisas, sou um ateísta cético.

Não. Concordo total e integralmente com tudo o que ele diz. Ateus não são arrogantes por esse fato. Podem ser, individualmente, por outras razões, de cada um. Crentes é que são arrogantes, por princípio, em razão de sua própria crença. Considerar que são seres especiais criados à imagem e semelhança de Deus é o cúmulo da arrogância.

Para o filósofo Michel Onfray, todo conhecimento, principalmente o científico, é uma ferramenta indispensável para o exercício da filosofia, caso contrário, o filósofo está "fora da realidade". Concorda?

Concordo plenamente. Para mim o filósofo tem que ser o polímata por excelência, especialmente com relação a conhecimentos de matemática, física, cosmologia, biologia, psicologia, neurociências, literatura, música, artes plásticas, sociologia, economia, religião, sem contar as subdisciplinas da própria filosofia, como ética, estética, lógica, política, metafísica, epistemologia. Realmente não dá para ser filósofo sem ter uma boa noção, e não apenas superficial mas de certa abrangência e profundidade desses assuntos.

Como pode haver a expansão dos gases da camada externa de uma estrela (na fase de gigante vermelha)? Parte do calor fornecido é transformado em trabalho, acarretando a expansão do gás?

Exatamente. O aquecimento do caroço promove a expansão do envelope, não só pelas colisões das partículas que ficaram mais energéticas, mas também pela pressão da radiação gama emanada do caroço. O sistema termodinâmico de uma estrela é extremamente mais complexo do que um gás, pois além de gás há plasma (gás ionizado) e radiação, todos interagindo, além de elétrons livres. Isso tudo leva o sistema a um comportamento termo-magneto-eletro-hidrodinâmico muito complexo, sem contar que o campo gravitacional e o seu gradiente são de grande relevância no sistema, que tem que ser tratado tanto quanticamente quanto relativisticamente quer sob o aspecto restrito quanto o geral. Há tratados sobre astrofísica nuclear relativística que abordam o assunto. Quando eu estiver em casa poderei passar algumas referências. Mas são livros em nível de doutorado.

Voce sabe alguma coisa sobre o Planeta Nove ou X? Ele realmente pode existir? O que acha? https://en.wikipedia.org/wiki/Planet_Nine

O planeta nove é uma hipótese a ser levada a sério para confirmação ou não. Meu palpite é de que não existe.
Leia, também, isto:
https://en.wikipedia.org/wiki/Planets_beyond_Neptune

Professor, me embolei em alguns conceitos aqui. Poderia diferenciar o Princípio da Incerteza e o Gato de Schödinger? Se possível, uma explicação breve seria de grande ajuda.

Explicações breves não explicam nada.
O princípio da incerteza estabelece que não é possível obter simultaneamente, com precisão, o valor de duas grandezas que medem propriedades de um sistema físico. Essas grandezas são ditas "conjugadas", como a posição e a quantidade de movimento (momento linear ou "momentum"), a energia e o tempo para medi-la, o momento angular e a posição angular e outras.

Aproveitando o gancho da outra pergunta,acredita que um dia a raça humana consiguirá frear o envelhecimento e alcançar a "imortalidade"?Em questão de conhecimentos científicos estamos distantes desse feito?

Acho que é possível sim, achar uma forma de interromper a senescência e prolongar indefinidamente a vida adulta, exceto, é claro, em caso de acidentes ou doenças infecciosas. Isso está ligado à preservação dos telômeros.
(NÃO CLIQUE: COPIE E COLE NA BARRA DO NAVEGADOR):
https://www.google.com.br/?=tel%C3%B4meros#q=tel%C3%B4meros
https://www.google.com.br/webhp?hl=pt-BR#hl=pt-BR&q=tel%C3%B4meros+e+envelhecimento
Todavia a obtenção dessa preservação ainda não está disponível e poderá levar décadas ou mais para ser alcançada.

ESCOLHA DO AMOR

Por que dizer que o amor é triste?
Triste é não ter amor nenhum!
Pois tendo amor, mesmo só um,
gáudio maior, se ter, não existe.
Mas há quem negue, amando, amar,
se quem lhe ama, amor também
ter, dedicado, a outro alguém,
sem um nem outro abandonar.
Ou quem impede o amor que tem
de amar, também, um tanto e quanto,
um outro amor, que lhe convém.
Para o amor jamais trazer
tristeza à vida de ninguém,
nunca, a só um, tem que escolher.

professor, se tenho uma carga Q, no interior de uma esfera, ela necessariamente será nula? Por ex: numa situação em que ela esteja a uma distância r do centro( r< raio) ela, também, será nula?

Não. O que acontece é que uma carga dentro de um corpo oco condutor não sofrerá força por parte de outras cargas situadas no corpo ou fora dele, em condições estáticas.

Professor, em lançamento oblíquo, velocidade escalar é o mesmo que a Vox? Meu professor colocou uma questão dizendo assim: Uma bola atirada com velocidade escalar de 6m/s. Eu adotei como a velocidade normal dela, Vó e não Vox. Estou errado?

Velocidade escalar é a velocidade e não uma componente sua, a não ser que, por acaso, a velocidade já esteja na direção do eixo em relação ao qual se quer a componente. Nesse problema, os 6 m/s são a velocidade de lançamento e não a sua componente horizontal. Você está certo.

Mas Ernesto, ainda sobre a densidade anômala da água. Por qual motivo, a água, algo tão essencial não só para vida marítima mas para a formação da nossa espécie e de todas as outras contém essa anomalia? Não seria uma forma de Deus preservar a vida nos mares e a própria cadeia alimentar?

Não. Os fatos naturais não têm propósito. O que acontece é que eles podem possibilitar ou não alguma ocorrência. Se possibilita, ela ocorre, se não, não ocorre. Se a dilatação anômala da água possibilita a sobrevivência de seres aquáticos onde a água congela, então isso acontece. Se não fosse assim, não haveriam seres aquáticos nessas condições. Mas isso não significa que essa propriedade existe "para que" os seres aquáticos possam existir nessas condições. Da mesma forma qualquer outra condição existente na natureza que possibilite a ocorrência de algo.

Quando colidimos com algo, parte da energia cinética é transformada em energia sonora e térmica (desconsiderando que energia seja um ente)?

Depende. Em uma colisão perfeitamente elástica (por exemplo entre ímãs que nem chegam a se tocar ao colidir) não há dissipação nenhuma de energia mecânica em térmica e acústica. Na interação, a energia cinética é convertida em potencial e depois volta a ser cinética novamente, sem nenhuma dissipação. Isso, contudo, é raro. Quase todas as colisões são parcialmente elásticas, de modo que parte da energia mecânica é convertida em som e calor.

Se a logica pode provar qualquer coisa, então ela poderia provar que a logica não pode provar qualquer coisa?

Mas a lógica não pode provar qualquer coisa. A lógica apenas pode provar aquilo que decorra, como conclusão, de um raciocínio corretamente encaminhado. Todavia a maior parte das constatações a respeito da realidade não são lógicas e sim evidências mostradas pela percepção que os sentidos têm do mundo, auxiliados, se preciso, por instrumentos, como lentes, microfones, medidores e vários outros. As assertivas que se fazem a respeito da estrutura e do comportamento da realidade, hauridas das evidências, não são produtos de raciocínios, não sendo, portanto, lógicas. Isso não as impede de serem verdadeiras, pois há dois critérios de verdade, a evidência e a prova. Todavia, toda prova, em última instância, advém de evidências primitivas, a partir das quais, por raciocínio, se tiram conclusões e das conclusões, por raciocínio, se tiram novas conclusões e assim por diante. Mas no princípio da cadeia de raciocínios sempre há constatações diretas da veracidade das assertivas obtidas por evidências, que não são lógicas.

Mesmo um corpo sofrendo aceleração constante irá variar sua velocidade?

Mas é claro, pois aceleração é, exatamente, variação da velocidade, exceto se for constantemente nula. Aceleração constante significa apenas que a velocidade sofre variações de igual magnitude em idênticos intervalos de tempo. Mas está sempre variando. Se não for constante, as variações de velocidade serão diferentes em idênticos intervalos de tempo.

Ernesto, quais são os fatos da física que te fazem pensar sobre a existência de Deus? Pois a densidade anômala da água me faz pensar muito sobre esse fato.

Os fatos da Física e de todas as ciências me fazem pensar exatamente na inexistência de Deus. Não vejo razão nenhuma para evocar alguma interveniência divina para nada do que acontece na natureza. A dilatação anômala da água é que permite que, no inverno, exista água líquida por baixo do gelo sólido que flutua sobre ela, não permitindo que os seres marinhos, lacustres e fluviais pereçam. Isso não indica nenhuma interveniência divina, simplesmente se não fosse assim eles morreriam e não existiriam peixes exceto onde o frio não congelasse a água.

Ernesto, meu professor de fisica1 pós uma questão de um elevador dando s(t), e sabendo q leva 4 segundos de uma parada a outra. Ele queria a velocidade do elevador. Ele resolveu derivando e botou t=4. Mas isso não está errado? Não seria a vel no t=4 isso? Primeiro eu igual v(t)=0 e achei t=3(cont1) Depois eu fiz a velocidade do elevador entre t=7 e t=3. Porque ele não pediu a velocidade do elevador no tempo 4. Ele só disse que leva 4 segundos de uma parada a outra é pergunta a velocidade. Eu não estou certo não?


Se o elevador se move de acordo com uma função s = s(t), sua velocidade média entre dois instantes intervalados de 4 segundos será:
= ( s(t + 4) - s(t) ) / 4
Derivar e achar a derivada em t = 4 realmente não é a velocidade média no intervalo considerado e sim a velocidade instantânea no instante 4. É preciso ler o texto da questão, exatamente da forma como foi proposta, para se interpretar o que foi, realmente, pedido.
View more

Como se coloca em ação um comunismo sem matar meia duzia de capitalistas? É impossível ! Até porque sabemos que o EUA na época, enviavam pessoas de Miami para cuba, em hotéis, e praticavam atos terroristas.

Claro que é possível se se entender que o atingimento do comunismo não é algo para ser obtido imediatamente, mas sim como o ápice de um processo evolutivo civilizatório ao longo de dezenas de gerações, o que demandará vários séculos. Exatamente por meio da educação para a solidariedade e o compartilhamento, para a abolição do individualismo, da ganância. Bem como com a eleição de candidatos comunistas (mas não socialistas estatizantes) que proponham e façam aprovar leis que distribuam gradativamente o capital das empresas por seus trabalhadores, que paulatinamente deixem de ser assalariados e se tornem sócios das empresas. Porque o que há de errado com o capitalismo não é o capital e sim a sua concentração e a existência de trabalhadores assalariados. Isso é que tem que acabar. Acabar com o proletariado e não com a burguesia, transformando todo proletário em burguês e capitalista. Não havendo ninguém que não seja capitalista e nenhum trabalhador assalariado, sendo o capital distribuído mais ou menos equitativamente por toda a população se está no comunismo (mas não no socialismo estatal). É preciso, também, gradativamente, ir diminuindo a ingerência do estado na economia e no governo até que ele desapareça por falta de necessidade de existir, acabando-se, assim, com as fronteiras e tornando todo o mundo uma única entidade politica, sem governo, o que é a anarquia, o suprassumo da ordem e da civilização. Sem polícia, sem leis, sem advogados, sem juízes, sem crimes, sem desonestidades, sem mando, sem obediência, sem exércitos, ser religiões, sem propriedade, sem dinheiro. Isso é perfeitamente possível e passível de ser atingido em alguns milênios. Sem nenhuma revolução e nenhum derramamento de sangue. Pacificamente.

Certa vez vi você dizendo que era viciado em coca cola se não me engano, e que bebeu a vida toda, você nunca teve problemas por isso? Eu adoro coca cola mas de tanto as pessoas falarem que faz mal eu deixei de tomar!

Bom, eu tomo Coca Cola Zero, por ser diabético. Antes eu bebia Coca Cola com açúcar e esse deve ter sido um fator que colaborou para eu adquirir o diabetes. Mas não só, pois em sempre comi muito doce que sempre preferi a qualquer salgado. Em festas eu sempre ia procurar as crianças para saber onde estavam os doces. Cerveja, eu detesto. Mas gosto de vinho e não do suave. Às vezes eu tomo cerveja, mas pouco e só das especiais, como Pale Ale. Nunca tomo cerveja pilsen dessas marcas nacionais.

Prof, vale a pena ler o the feynman's lectures on computation ou é muito desatualizado?

Não conheço esse livro. Conheço "The Feynman Lectures on Physics" que foi traduzido para o português como "Lições de Física de Feynman", que é muito bom mesmo para um livro complementar ao livro texto de Física Geral de um Curso de Física, especialmente o "Curso de Física de Berkeley" ou o "Alonso & Finn". Mas não é um livro para um curso de engenharia.

Homem que não namora mulher mãe solteira é machista?

Não necessariamente. É muito errado aplicar rótulos a pessoas em razão de alguma concepção, comportamento, atitude, opinião ou algo do tipo isoladamente. Há que se considerar o conjunto de suas concepções, seu comportamento global, suas atitudes em geral e o grosso de suas opiniões. Então se pode dizer que essa pessoa seja machista, feminista, esquerdista, direitista, teísta, ateísta, racista, globalista, capitalista, comunista, socialista, anarquista ou outro "ista" qualquer.

Esta refutação do materialismo é aplicável ao fisicalismo? O que pensas sobre? http://blog.criticanarede.com/2010/04/refutacao-do-materialismo.html

Não é verdade que real só seja o que não depende de nada para existir, ou seja, que existe por si mesmo. Leibniz erra. Real é o que existe no mundo exterior a nossas mentes, mesmo que sua existência seja dependente de algo para acontecer. Portanto o que se segue, isto é, que algo real não possa ter partes, que não tenha extensão espacial e que, de acordo com essa concepção, não possa ser material, está inteiramente errado. Qualquer coisa real pode ter partes sim e pode ser material ou física sem ser material (como os campos e a radiação). Quanto a alguma entidade espiritual, essas sim, é que não são reais, pois, ao contrário, só existem como concepções dentro de mentes e não no mundo objetivo exterior às mentes.

quanto ganha um fisico no brasil?Estudar fisica no bacharelado toma muito tempo na vida do estudante?Teria como paralelamente estudar economia de maneira autodidata?

Um físico geralmente vai trabalhar em um instituto de pesquisa ou em uma universidade. Como ele tem que ter, pelo menos, o doutorado para entrar (se passar nos concursos), seu salário vai ser o de um professor adjunto com dedicação exclusiva. Veja esta tabela (terceira tabela):

Professor, o que acha do Che Guevara? Essas historia de que ele é um assasino etc, como se os EUA não fosse, haha.

São fatos independentes. Os Estados Unidos são, como nação, oficialmente, assassinos, do mesmo modos que Cuba, o Reino Unido, a França, a Itália, a Alemanha, o Brasil, a Argentina, a Rússia, a China, o Irã, o Iraque, a Síria, Portugal, Espanha, o Japão e praticamente todos os países do mundo. Pessoalmente Che Guevara, Fidel Castro, Pinochet, Hitler, Stálin, Mao, Pol Pot e muitos outros foram e são assassinos também. Assassinatos por um lado não justificam assassinatos por outro lado.

Mestre o que acha sobre ser vegetariano? Poderíamos viver sem comer carne? Ou a carne é essencial pra nossa alimentação?

Pode-se perfeitamente viver sem comer carne. A carne não é essencial para a alimentação. Além de seu consumo acarretar o assassinato em massa de muitos seres sencientes, o que é um crime. Mesmo sendo o ser humano onívoro, ele não necessita consumir carne, como acontece com os exclusivamente carnívoros.

Qual a melhor forma de se aprender programação? por qual linguagem começar?

Experimente a Phyton. O jeito de aprender é pegar um livro daqueles de mil páginas sobre a linguagem e ir estudando página por página, da primeira até a última, enquanto vai treinando em um computador.
https://pt.wikipedia.org/wiki/Python
https://www.google.com.br/search?tbm=bks&q=python

Até que idade o QI e o QE de uma pessoa pode vir a se desenvolver?

Não há um limite. Quanto mais novo, mais os enfrentamentos de situações e desafios colaboram para aprimorar a inteligência, que, em parte, pode ser indicada por esses índices. Especialmente na tenra infância, até uns quatro anos de idade. Mas depois disso, também se consegue aprimorar todos os tipos de inteligência se se dedicar com afinco e sem preguiça a vencer os obstáculos cognitivos, operativos, relacionais e de toda ordem que se apresentarem. Isso vale para a vida toda, mesmo na velhice. Mas essa capacidade, certamente, diminui com o avançar da idade. Todavia, em compensação, quem passou a vida toda estimulando a inteligência, adquiriu um cabedal que facilita muito se galgar mais degraus nesse aprimoramento.

Professor. Embora eu seja ateu como você, considero que existem diversos bons ensinamentos em algumas religiões. Atualmente, considero o Dalai Lama como uma das figuras mais sensatas entre os religiosos do mundo. Seus pensamentos são todos voltados para a bondade. Concorda comigo?

Sem dúvida há muito que seja proveitoso nas religiões, especialmente o seu trabalho caritativo e de apoio e conforto das pessoas em suas vicissitudes. Todavia isso pode muito bem ser feito sem as religiões, por meio de uma atividade humanista puramente secular. O problema é que as religiões, ao fazerem isso, remetem à intervenção de entidades fictícias, às quais as pessoas acabam creditando os resultados benéficos obtidos, mas não os maléficos ou os inócuos. Nisso é que consiste a maldade das religiões. Os ensinamentos éticos do Dalai Lama são excelentes, todavia eles são inseridos no contexto de se livrar do ciclo de sansara, o que é inteiramente ilusório. O budismo, sem os aspectos supranaturais, é uma boa escolha de religião. Mas o que ele tem de bom, como o que as demais também têm, pode perfeitamente ser cultivado em uma concepção de espiritualidade puramente humanística, que não inclua as noções de divindades, espíritos e nem de alma imortal. Nesse sentido recomendo a leitura dos livros "Despertar", de Sam Harris, "O Espírito do Ateísmo", de Andre Comte-Sponville e "Religião para Ateus" de Alain de Botton.

LinkWithin

Related Posts with Thumbnails